Сложные логические вопросы: Сложные загадки на логику — логические задачи с ответами

Содержание

Сложные загадки на логику — логические задачи с ответами

Логическая загадка любой сложности заставит вас подумать о давно забытых предметах, вспомнить содержание детских сказок и смириться с тем, что ваш ребенок находит ответы быстрее вас. Хотите проверить? Мы подготовили специальную подборку сложных логических загадок!

Зачем решать логические загадки?

При решении логических задачек и поиске «отгадки» у человека начинают работать оба полушария мозга. Левое полушарие отвечает за логику и стремится разобраться в причинных связях. Правое несет ответственность за интуицию, учится строить целостную картину и формировать образное мышление.

У современных людей левое полушарие включается в работу не так часто — «загуглили» и отправились по делам. И, конечно, свою “логическую мышцу” можно и нужно тренировать — тут нам и пригодятся загадки на логику – они помогают держать мозг в тонусе даже в самые «ленивые» дни.

Эффект от «логических тренировок» даст о себе знать уже после первых занятий — натренированный мозг быстрее соображает, позволяя решать не только математические задачи, но и находить выход из жизненных ситуаций.

Зачем детям развивать логику?

Развитое логическое мышление поможет вашему ребенку легко справляться с математическими задачами не только в начальной, но и в старшей школе.

Умение анализировать выручит его на литературе, а способность нестандартно мыслить пригодится в творческих кружках и школьных активах. И, конечно, он всегда сможет дать аргументированный ответ на поставленный учителем каверзный вопрос, а еще переспорит одноклассников и точно завоюет авторитет в школе!

Боитесь, что ваш ребенок не справится, потому что «гуманитарий»? В Умназии вы найдете сотни загадок на логику, которые адаптированы для детей разных классов и возрастов, которые позволяют развивать Логику постепенно.

Не отказывайтесь от «мозговых тренировок» — начните мыслить не так, как все!

Сложные загадки на логику

Давайте попробуем разгадать 5 загадок на логику. Дети и взрослые могут размышлять над ответом вместе.

Загадка №1

Каких камней не бывает в речке?

Показать ответ

Ответ: В речке вы никогда не найдете сухих камней. А вот драгоценные попасться могут 🙂

Загадка №2

На столе лежат две монеты, в сумме они дают 3 рубля. Одна из них — не 1 рубль. Какие это монеты?

Показать ответ

Ответ: На столе лежат 2 рубля и 1 рубль. В условии сказано, что только одна из монет – не рубль.

Загадка №3

Что не вместится даже в самую большую кастрюлю?

Показать ответ

Ответ: Крышка этой кастрюли 🙂 Не ходите на кухню и не проверяйте – точно застрянет!

Загадка №4

Что может в одно и то же время стоять и ходить, висеть и стоять, ходить и лежать?

Показать ответ

Загадка №5

Завязать можно, а развязать нельзя. Что это такое?

Показать ответ

Ответ: Нет, не шнурки от старых кроссовок. Правильный ответ — разговор.


Справились? Если да – вы молодцы. А если что-то не получилось – приходите в Умназию и тренируйте ум с умом!

Умназия – образовательная онлайн-платформа для учеников начальной школы. На платформе ребенок сможет:
Развить логику и внимание на тренажере навыков, прокачать память и стать финансово грамотным!

У нас ребенку не будет скучно: умные алгоритмы подготовят для него индивидуальную программу, а в онлайн-тренажере он будет раскрывать тайны, получать достижения и становиться героем Умназии.

А для самых умных и мотивированных ребят у нас проводятся олимпиады по 4 предметам, которые готовят детей к будущим олимпиадам в школах. Приходите и развивайтесь вместе с нами!


Теперь приступим к более сложным заданиям. Включайте логику и начинайте!

Загадка №6

Я – вода, и по воде плаваю. Кто я такая?

Показать ответ

Ответ: Льдина.

Загадка №7

Ползут 3 черепахи.1-я черепаха говорит: за мной ползут две черепахи.
2-я черепаха говорит: за мной ползёт одна черепаха и передо мной ползёт одна черепаха. А 3-я черепаха: передо мной ползут две черепахи, и за мной ползёт одна черепаха.

Как такое может быть?

Показать ответ

Ответ: Черепахи ползут по кругу!

Загадка №8

На ферме было 2 коня, 1 кролик, 1 щенок, 1 кошка, свинья и поросенок, корова и теленок, индюк и гусь.
Пришел хозяин с собакой. Сколько на ферме стало ног?

Показать ответ

Ответ: 24. Почему? Потому что ноги только у человека, лошадей, свиней и коров. У остальных животных — лапы.

Загадка №9

Где впервые был обнаружен картофель?

Показать ответ

Ответ: Отгадка предельно простая – в земле.

Загадка №10

Странный дождь порой идет: сотней струй он кверху бьет.

Показать ответ

Ответ: Фонтан.


Почему логические загадки полезны для детей? Они развивают смекалку, учат работать с информацией, «прокачивают» логико-математический интеллект и делают ребенка более самостоятельным.

Помимо успехов в школе, о которых мы уже говорили, развитое логическое мышление способствует формированию уверенности в себе в процессе повседневной жизни.

>25 тысяч учеников уже решают авторские задачи в Умназии!


А теперь продолжим тренироваться, ведь долго без логики не протянешь!

Загадка №11

Скажешь «не приходи!» — всё равно приходит. Скажешь «не уходи!» — всё равно уходит. Что это такое?

Показать ответ

Загадка №12

Вы сидите в самолете, впереди вас лошадь, сзади автомобиль. Где вы находитесь?

Показать ответ

Ответ: Вы катаетесь на карусели.

Загадка №13

Чем больше из нее берешь, тем больше она становится. Что это?

Показать ответ

Загадка №14

Что принадлежит вам, однако другие этим пользуются чаще, чем вы сами?

Показать ответ

Ответ: Ваше имя.

Загадка №15

Сколько яиц можно съесть натощак?

Показать ответ

Ответ: нет, размер вашего аппетита тут не при чем. Натощак можно съестьскушать только 1 яйцо, потому что все следующие будут съедены уже не на пустой желудок.

На сегодня это все — мы с вами славно потрудились.

Не хотите останавливаться на достигнутом? Регистрируйтесь в на нашей платформе и решайте тысячи задач в онлайн-режиме. Создавайте индивидуальный учебный план для вашего ребенка и развивайте пять навыков в удобное время.

Начните заниматься с ребенком уже сегодня!

Развитие логики у детей 7-10 лет

Развиваем критическое нестандартное мышление, учим работать с информацией и принимать верные решения

узнать подробнее

Читайте также:


 

Загадки на логику с подвохом

Решение различных задач и загадок, способствующих развитию логического мышления, само по себе сложное и затягивающее в «омут знаний» занятие. А когда задачки еще и с подвохом, думать тяжелее вдвойне: ответ на вопрос может оказаться предельно простым и ввести в «ступор» даже бывалого математика.

Не нервничайте раньше времени и подключите к решению загадок детей, а еще лучше – зайдите с ними на сайт Умназии и составьте индивидуальную программу. Развивайте пять навыков мышления вместе с нами и будьте уверены в том, что ваш ребенок справится с любой логической (и даже алогической) школьной задачей.

Загадки с подвохом. В чем суть?

Вопросы с «подковыркой» содержат «тайну», которую нужно разгадать и детям, и взрослым. Здесь нет очевидных ответов, «прозрачных» решений и скучных алгоритмов. Верный ответ порой переворачивает сознание с ног на голову и заставляет взглянуть на мир по-другому. Дети учатся мыслить нестандартно и расширяют кругозор, взрослые смеются и (или) хватаются за голову в бесконечных попытках найти правильное решение.

Зачем это нужно?

Когда ребенок решает обычную задачу, он ищет самый «логичный» ответ на нее. Учится думать и анализировать, обрабатывать информацию, оценивать факты и делать выводы. При решении загадок с подвохом ему приходится менять алгоритм и подключать к работе не только левое «аналитическое» полушарие, но и «творческое» правое, которое оперирует вдохновением и фантазией.

Найти правильный ответ помогает и то, и другое: в загадках с подвохом важно не только соотнести аргументы, посылки и выводы, но и предугадать неоднозначность самой отгадки.

Загадки с подвохом для детей

Начнем с малого и решим 5 загадок для самых маленьких. Взрослым ответ покажется очевидным, а вот ребятам придется подумать (но может быть и наоборот). Чур, не подсказывать!

Загадка №1
Что ты никогда не сможешь съесть на завтрак?

Показать ответ

Ответ: Обед и/или ужин.

Загадка №2.
Что может путешествовать по свету, оставаясь в одном и том же углу?

Показать ответ

Ответ: Почтовая марка.

Загадка №3
В одноэтажном розовом доме жил розовый человек, розовый кот, розовая рыбка, был розовый компьютер, розовое кресло, розовый стол, розовый телефон, розовая душевая кабина – все было розовым! Какого цвета была лестница?

Показать ответ

Ответ: В этом доме не было лестницы, потому что это был одноэтажный дом.

Загадка №4
Что настолько же огромное, как слон, но ничего не весит?

Показать ответ

Ответ: Тень слона.

Загадка №5
Какое слово в словаре написано неправильно?

Показать ответ

Ответ: Это слово «неправильно».

Мы знаем, что вам и вашим детям, как минимум, было интересно. А как максимум – вы серьезно задумались и хотите идти дальше!

СМЕШНЫЕ ЗАГАДКИ НА ЛОГИКУ

Подойдут для всех возрастных категорий и потребуют максимально нестандартных решений. Не заставят скучать, рассмешат и научат мыслить логически даже самого нелогичного человека. Переходим к изучению и поиску правильного ответа!

Загадка №1
У каких волос есть сходство с океаном?

Показать ответ

Ответ: У волнистых.

Загадка №2
Мужчина гулял в парке, когда на улице пошел дождь. У мужчины не было зонтика, и он не мог накинуть на себя плащ или накрыться шляпой. На нем не осталось сухой одежды, однако ни один волос на его голове не промок. Как это могло произойти?

Показать ответ

Ответ: Мужчина был лысый.

Загадка №3
Какой рукой удобнее размешивать сахар в стакане с чаем?

Показать ответ

Ответ: Рукой, которая держит ложку.

Загадка №4
На что больше всего похожа половинка яблока?

Показать ответ

Ответ: На вторую половинку этого же яблока.

Загадка №5
Какое изобретение позволяет смотреть сквозь стены?

Показать ответ

Ответ: Ничего сверхъестественного. Сквозь стены мы смотрим через окно.

Посмеялись? Если да, мы вас поздравляем: ученые утверждают, что смех продлевает жизнь. Если нет, не грустите – попробуйте еще раз вчитаться в условие и соотнести его с правильным ответом. Даже если после этого на вашем лице нет и намека на улыбку, будьте уверены, вы старались не зря: развитое логическое мышление непременно пригодится в повседневной жизни вам и вашему ребенку.

Трудные загадки с подвохом

Нет, мы еще не закончили. Последние пять загадок будут особенно сложными: здесь детям понадобится помощь взрослых. Опять придется мобилизовать оба полушария мозга, немного понервничать и вспомнить кое-что из школьного курса обществознания. Поразмышляйте над ответом, «пораскиньте» мозгами и найдите то самое решение!

Загадка №1
Человек выпрыгнул из самолёта без парашюта. Он приземлился на твёрдый грунт, но остался невредимым. Почему?

Показать ответ

Ответ: Самолёт стоял на земле.

Загадка №2
«Мой брат – адвокат», — говорит бухгалтер. Адвокат говорит, что у него нет брата. Кто из них лжет?

Показать ответ

Ответ: Никто, потому что бухгалтер – сестра адвоката.

Загадка №3
Мужчина ехал по дороге на своем грузовике. Фары не были включены. Луна не светила. Перед грузовиком женщина в черной одежде переходила дорогу. Догадайтесь, как мужчина ее увидел.

Показать ответ

Ответ: Женщину было отчетливо видно, потому что это происходило днем, а не ночью.

Загадка №4
Однажды за завтраком девушка уронила кольцо в чашку с кофе. Однако кольцо осталось сухим. Почему?

Показать ответ

Ответ: Кофе не был напитком: он был в зернах. Если вы ответили, что кофе был молотый или растворимый, вы тоже правы.

Загадка №5
Что все время приходит, но никак не придет?

Показать ответ

Ответ: Завтра.

Ура! Мы с вами поработали на отлично и стали чуть-чуть умнее.


Хотите еще?
Приводите ребенка в Умназию и развивайтесь вместе с нами.

В вашем распоряжении более 4000 авторских задач, выбор траектории обучения, игровой формат и формирование каждого из пяти навыков. Логика, эрудиция, внимание, память и обучаемость – настраивайте и выбирайте наиболее интересные для ребенка задания, получайте награды и вместе радуйтесь новым победам.
Начните заниматься в Умназии уже сегодня!

Развитие логики у детей 7-10 лет

Развиваем критическое нестандартное мышление, учим работать с информацией и принимать верные решения

узнать подробнее

Читайте также:


 

Смешные загадки для детей — смешные загадки на логику с ответами

Ребенок хочет отдохнуть от школы, а вы боитесь, что он «расслабится» и перестанет учиться? Предложите ему отгадать пару смешных загадок на логику – он получит удовольствие, а заодно «прокачает» пять навыков мышления и научится отвечать на самые каверзные вопросы.

Зачем решать задачи на логику?

Поиск отгадки – своеобразная тренировка для мозга. В процессе размышлений ребенок учится анализировать и оценивать факты, грамотно обрабатывать информацию и мыслить не по шаблону. Найденное решение формирует успех и уверенность в себе, а игровой формат обучения не дает заскучать и отвлечься. Ребенок становится собранным, усидчивым и находчивым, быстрее усваивает новое и начинает по-настоящему радоваться победам.
Шуточные загадки помогают школьнику расслабиться, взрослым – снять стресс и от души посмеяться. Если чувствуете усталость — познакомьтесь с подборкой, которую мы составили специально для вас.

Смешные загадки для детей

Загадка №1.
Задание: Воробей может съесть горсточку зерна, а лошадь не может. Почему?

Показать ответ

Ответ: Воробей слишком маленький, чтобы съесть лошадь.

Загадка №2.
Задание: За что учеников выгоняют из класса?

Показать ответ

Ответ: За дверь.

Загадка №3.
Задание: Какое колесо не крутится при правом развороте?

Показать ответ

Ответ: Запасное.

Загадка №4.
Задание: Какой конь не ест овса?

Показать ответ

Ответ: Шахматный конь.

Загадка №5.
Задание: Как каплю превратить в цаплю?

Показать ответ

Ответ: Заменить букву «к» на «ц».

Отлично! А теперь отодвиньте от экрана детей – мы загадаем загадки только для взрослых.

Смешные загадки для взрослых

Загадка №1.
Задание: Что нужно делать, если вы сели в машину, а ноги до педалей не достают?

Показать ответ

Ответ: Просто пересядьте на водительское кресло.

Загадка №2.
Задание: Какая разница между попом и Волгой?

Показать ответ

Ответ: Поп — батюшка, а Волга — матушка.

Загадка №3. «Не Бенджамин Баттон».
Задание: Я – высокий, когда молодой. Я – маленький, когда старый. Что я такое?

Показать ответ

Ответ: Свечка.

Загадка №4.
Задание: Не лёд, а тает, не лодка, а уплывает.

Показать ответ

Ответ: Зарплата.


Посмеялись? Отлично! Мы уверены, что смех продлевает жизнь. А если вы слишком серьезны, загляните на сайт Умназии —найдете 3484 авторские задачи, и какие-то из них вам обязательно приглянутся. И, конечно, захватите с собой детей — в Умназии они смогут развивать пять навыков мышления, участвовать в олимпиадах, зарабатывать личные достижения и самостоятельно выбирать траекторию обучения. Приходите в Умназию — у нас не бывает скучно.

Развитие логики у детей 7-10 лет

Развиваем критическое нестандартное мышление, учим работать с информацией и принимать верные решения

узнать подробнее

Читайте также:


 

7 самых сложных логических задач, которые решит только один человек из десяти

7 самых сложных логических задач, которые решит только один человек из десяти

В связи с началом учебного года мы решили проверить, насколько наши подписчики умны и изобретательны. А ты сможешь решить все, представленные нами, задачи?

«ПОСЧИТАЙ-КА»

Давай проверим, умеешь ли ты считать?

Реши без помощи калькулятора вот этот пример: К 1000 нужно прибавить 40, потом еще 1000. Затем приплюсуйте 30. Есть? Теперь снова 1000. Добавьте 20. Еще раз 1000. И напоследок 10.

Сколько получилось?

А теперь проверь все еще раз с помощью своего телефона. Совпало?

«ЧТО БОДРИТ УТРОМ?»

А теперь задачка на логику.

Женщина уронила в стакан, полный кофе, свой перстень. Как он мог остаться сухим?

Как ты думаешь, в чем тут секрет?

«СПИЧКИ ДЕТЯМ НЕ ИГРУШКА»

Сколько спичек на картинке?

«ЗЕЛЕНЫЙ ЧЕЛОВЕЧЕК»

Это та загадка, которую ты решишь с помощью детской наивностью. Мы уверены, её можно отгадать с первого раза! Ответь на вопрос: что нужно сделать, когда видишь зеленого человечка?

Учитель рисует на листке бумаги несколько кружков и спрашивает одного ученика: «Сколько здесь кружков?». «Семь» — отвечает ученик. «Правильно. Так сколько здесь кружков?» — опять спрашивает учитель другого ученика. «Пять» — отвечает тот. «Правильно» — снова говорит учитель. Так сколько же кружков он нарисовал на листке?

Думаешь все так легко? А теперь попробуй решить задачи, которые считаются самыми сложными в мире!

«СУПЕР СУДОКУ»

Первое, над чем мы предлагаем тебе поломать голову – это самая сложная судоку в мире.

Судоку – это японская головоломка с числами. Принцип ее совсем не замысловат. Но ту, которую предложили тебе мы, сможет решить точно не каждый!

«БОГИ ЛОГИЧЕСКИХ ЗАДАЧ»

Есть три бога, A, B, и C, один из которых бог истины, другой бог лжи и третий бог случая, причём неясно, кто из них кто. Бог истины всегда говорит правду, бог лжи обманывает, а бог случая может сказать и то, и другое в произвольном порядке. Необходимо определить, кем является каждый из богов, задав три вопроса, на которые можно ответить «да» или «нет», при этом каждый вопрос задаётся только одному богу. Боги понимают вопросы, но отвечают на своём языке, в котором есть слова «da» и «ja», но неизвестно, какое слово обозначает «да», а какое «нет».

Эта логическая задача за авторством американского философа и логика Джорджа Булоса была впервые опубликована в итальянской газете «la Repubblica» в 1992-м году. Так же в загадке есть комментарии создателей:

– Можно задавать одному богу более чем один вопрос (поэтому другим богам может быть не задано ни одного вопроса вообще).

– Каков будет следующий вопрос и кому он будет задан, может зависеть от ответа на предыдущий вопрос.

– Бог случая отвечает случайным образом, зависящим от подбрасываний монетки, спрятанной в его голове: если выпадет аверс, то отвечает правдиво, если реверс — то врёт.

– Бог случая отвечает «da» или «ja» на любой вопрос, на который можно ответить «да» либо «нет».

Ответы на все задачи можно посмотреть по ссылке.

Нашли опечатку в тексте? Выделите её и нажмите ctrl+enter

ЗАГАДКИ НА ЛОГИКУ — самые СЛОЖНЫЕ!

Загадка: Знаете ли вы сколько в одном году на самом деле лет? Ответ: Столько же, сколько зим — одно

Загадка: Что все время пребывает в покое, даже когда поднимается или спускается с холма? Ответ: Дорога

Загадка: Долго ли следует варить в кипящей воде крутое куриное яйцо? Ответ: Это бессмысленное дело (яйцо уже сварено)

Загадка: Когда барану исполнится восьмой год, что будет? Ответ: Пойдёт девятый

Загадка: В каком случае 15 стариков и 15 внуков, могут попытаться спрятаться под один и тот же зонтик, совершенно при этом не вымокнув? Ответ: Только в одном – при полном отсутствии осадков

Загадка: Какое автомобильное колесо не поворачивается вместе со всеми, когда водитель крутит баранку руля? Ответ: То, что в запаске

Загадка: Когда индюк стоит на двух лапах, его вес составляет порядка 6-ти килограмм. Каким станет вес индюка, если встанет не на две, а только на одну лапу? Ответ: Он не изменится и будет по-прежнему 6 килограмм

Загадка: В центре озера на острове стоит тюрьма. Как-то раз из нее решили удрать два заключенных. Первому не повезло: когда он переплывал по озеру, его заметила береговая охрана. Охранники схватили его за длинные волосы и затащили на лодку. Второму повезло еще меньше: в озере его сожрала акула. Найдите в этом рассказе две лжи. Ответ: Первая ложь: все заключенные – лысые. Вторая ложь: в озерах не водятся акулы.

Загадка: Четверо мальчишек гоняли мяч во дворе своего дома и разбили одно из окон. Из него выглянула женщина и строго спросила: «Кто из вас разбил мое окно?». Валера сразу же сказал: «окно разбил Сергей или Артем». Сергей парировал: «Я его не разбивал!». Максим сказал: «Его разбил Артем». Артем ответил последним: «Максим не прав». Кто же разбил окно, если учитывать, что трое мальчиков сказали правду? Ответ: Артем.

Загадка: У мэра города N есть родной брат Константин. Но у Константина брата нет! Как такое может быть? Ответ: мэр города N является женщиной.

Загадка: Две девочки решили посчитать в парке прохожих. Одна сидела на лавочке и считала всех, кто проходил по аллее мимо нее. А другая девочка ходила по аллее вперед и назад, и считала только тех, кто шел её навстречу. Какая девочка насчитала больше прохожих: первая или же вторая? Ответ: они насчитали одинаковое количество прохожих.

Загадка: Кто способен без труда зажечь под водой спичку? Ответ: Матрос на подлодке

Загадка: Когда сидишь, что невозможно взять в руки? Ответ: стул, на котором сидишь.

Загадка: Чтобы начать его использовать его необходимо разбить. Ответ: яйцо. Его используют для приготовления пищи.

Загадка: У Петиного отца было четыре сына: Александр, Михаил, Иван и… Кто четвертый? Ответ: Петр.

Загадка: Один волшебник сказал, что запросто может в центре комнаты поставить пустую бутылку и вползти в нее. Как он это сделает? Ответ: волшебник запросто может вползти в нее – в комнату.

Загадка: Какой объем земли содержится в яме, диаметр которой 5 метров, а глубина – 4 метра? Ответ: яма пустая – в ней ничего не содержится.

Загадка: Зачем вода в стакане? Ответ: за стеклом.

Загадка: Сколько нужно кирпичей, чтобы полностью завершить постройку дома? Ответ: один последний.

Загадка: Какими нотами можно измерить любое расстояние?

Загадка: Что общего между числами 69, 88? Ответ: в перевернутом виде они выглядят также.

Загадка: Эскимосы являются отличными охотниками, но почему они никогда не охотились на пингвинов? Ответ: Пингвины обитают на Южном полюсе, эскимосы же живут на Северном.

Загадка: Невесома, однако бывает и тяжелой, и легкой. О чем речь? Ответ: музыка.

Загадка: Кого Господь никогда не видит, король видит очень редко, а обычный человек каждый день? Ответ: себе подобных.

Загадка: Кто ходит сидя? Ответ: игрок в шахматы.

Загадка: В какой части планеты могут дуть только северные ветра? Ответ: на Южном полюсе.

Загадка: Ночной охранник умер в дневное время. Сможет ли он рассчитывать на пенсию? Ответ: Мертвый сторож не нуждается в пенсии.

Загадка: Как достичь того, чтобы пять рабочих оказались в одном ботинке? Ответ: попросить каждого рабочего снять по одному ботинку.

Загадка: Дикий кабан залез на сосну с четырьмя лапами, а спустился – с тремя. Как такое может быть? Ответ: кабаны не могут лазать по деревьям.

Загадка: Мальчик зашел в хлев и увидел там пять коров, три свиньи, две лошади и одну кошку. Сколько всего ног в хлеву? Ответ: две (ноги мальчика), так как у животных – лапы.

Загадка: У Бога она есть, а у короля – нет. У бобра она спереди, а у верблюда – посредине. Что же это? Ответ: буква «Б».

Загадка: В семье негров в Конго родился ребенок: весь белый, даже зубы были белоснежными. Что здесь не так? Ответ: дети рождаются без зубов.

Загадка: Человек уронил монету в чашку с чаем. Как ему достать монету, не намочив пальцев? (Пить чай при этом нельзя). Ответ: чай в чашке не заваренный.

Загадка: Она может постареть всего за пару часов. Она приносит людям пользу, убивая при этом себя. Ветер и вода могут спасти её от гибели. Что это такое? Ответ: свечка.

Загадка: Что может быть теплее одного пухового одеяла? Ответ: два пуховых одеяла.

Загадка: Какой из месяцев в году самый длинный? Ответ: сентябрь (целых 8 букв).

Загадка: Что очень напоминает собой половинка инжира? Ответ: вторую половинку инжира.

Загадка: В каком месяце в году человек спит меньше всего? Ответ: в феврале (самый короткий месяц).

Загадка: Какой станет белая майка, если бросить её в Красное море? Ответ: она станет мокрой.

Загадка: Какой продукт можно варить хоть в килограмме соли, а он все равно окажется не соленым? Ответ: яйца.

Загадка: Завтра это будет вчера, а вчера это было завтра. Что это такое? Ответ: сегодня.

Загадка: Каким образом можно переносить воду в дуршлаге? Ответ: если предварительно его заморозить.

Загадка: Из какого стакана нельзя выпить воды? Ответ: из пустого.

Загадка: На улице шел дождь. По городу ехал троллейбус. В нем было всего шесть пассажиров, и все они спали. Не спал лишь водитель. Какие окна были в водительской кабине? Ответ: мокрые.

Загадка: На каких часах можно посмотреть правильное время лишь два раза за сутки? Ответ: на поломанных.

Загадка: Что никогда нельзя уместить в самую гигантскую кастрюлю? Ответ: её крышку.

Загадка: Сколько груш можно скушать натощак? Ответ: Одну грушу (все прочие груши будут уже не натощак «поглощаться»).

Загадка: Чем заканчивается и февраль, и ноябрь? Ответ: мягким знаком.

Загадка: Она у Артема впереди, а у Тамары сзади? Ответ: «а».

Загадка: Что изменяется во внешнем облике супругов после свадьбы? Ответ: у них появляются обручальные кольца.

Загадка: Сколько будет 5+5*5=? Ответ: 30, начинаем с умножения.

Загадка: Если в доме все окна на север, какое животное может постучаться в окно? Ответ: Пингвин (только на южном полюсе все 4 стороны на север).

Загадка: Какую болезнь нельзя подхватить на земле? Ответ: Морскую.

Загадка: Можно ли спрыгнуть с десятиметровой лестницы, не разбиться и даже не ушибиться? Ответ: можно, если спрыгнуть с самой низкой ступеньки.

Загадка: У каждого из шести братьев есть по три сестры. Сколько всего сестёр? Ответ: три.

Загадка: По чему можно ходить, бегать, ползать, но практически невозможно ездить? Ответ: по лестнице.

Загадка: В Москве — 6, в Костроме — 8, в Вологде — 7. О чём идет речь? Ответ: о том, сколько букв в названии города.

Загадка: Москва стоит 100, Архангельск стоит 500, Ярославль стоит 1000. О чём идет речь? Ответ: посмотри на российские рубли и все поймешь.

Загадка: Что расположено между селом и городом? Ответ: «И».

Загадка: Можно ли что-то увидеть с плотно закрытыми глазами? Ответ: можно — сон.

Загадка: У трёх врачей есть брат Игорь, но у самого Игоря братьев нет и не было. Может ли такое быть? Ответ: Да. три врача — сестры Игоря (слово врач всегда мужского рода, независимо к кому оно относится к мужчине или женщине).

Загадка: Они могут одновременно висеть, идти и отставать, стоять, ходить и спешить, лежать, идти и врать. Что это? Ответ: Часы.

Загадка: В круглой комнате в каждом углу сидит по две пантеры, а напротив каждой из них по три пантеры. У каждой из пантер на хвосте сидит кошка. Сколько кошек сидит в этой комнате? Ответ: 0 — в круглой комнате нет углов.

Загадка: У вас в руках белый шелковый платок, а сами вы стоите на берегу Красного моря. Что произойдет с вашим платком на дне Красного моря, если вы будете иметь несчастье уронить его туда? Ответ: платок намокнет.

Загадка: Есть ли шанс где-либо отыскать прошлогодний снег, и если можно, то где его найти? Ответ: в новогоднюю ночь после 12 часов ночи весь снег прошлогодний.

Загадка: В лесу растет дикая слива, а на ней — яйца? Выдумка это или нет? Ответ: на любом дереве могут быть птичьи яйца (в гнезде).

Загадка: Если вы съедите вкусный свежеиспеченный бублик, то что при этом вы потеряете? Ответ: потеряем чувство голода.

Загадка: Главное отличие лошади от иглы? Ответ: на иглу сначала сядешь, а потом как подпрыгнешь, а на лошадь сначала нужно подпрыгнуть, а потом только сесть.

Загадка: Тяжкая доля у этого предмета. Тот, кто в нем остро нуждается – бросает его, а когда в нем и нужды то нет, его поднимают и берут с собой. О каком предмете идет речь? Ответ: о якоре.

Загадка: У окна стояли зажженными пять свечей, четыре из них погасили хозяева пред уходом. Сколько свечей осталось в итоге? Ответ: остались четыре свечи, так как погашенные свечки не сгорят полностью в отличие от той, что осталась зажженной.

Загадка: Стороны крыши сарая не имеют симметрии. Правая сторона построена под углом 65 градусов, а левая – под углом 80 градусов. На крышу сарая забрался гусак, и снес яйцо весом 50 грамм прямо на гребень крыши сарая. На какую сторону крыши упадет яйцо? Ответ: гусаки не откладывают яйца.

Загадка: В саду, на апельсиновом дереве зреет 92 плода. Ночью дул очень сильный ветер и шел дождь, к утру с дерева упали 35 мандаринов. Сколько же плодов осталось висеть на дереве? Ответ: на апельсиновом дереве осталось 92 плода, мандарины упали с мандаринового дерева.

Яндекс Дзен | Открывайте новое каждый день

Яндекс Дзен | Открывайте новое каждый день

Яндекс.Дзен – это платформа, которая подбирает контент специально для вас. В Дзене есть статьи и видео на разные темы от блогеров и медиа.

Ваш личный Дзен

Дзен понимает ваши интересы и собирает ленту для вас. Он анализирует действия: что вы смотрите, кому ставите лайки, на кого подписываетесь, а после – рекомендует вам и уже любимые источники, и ещё неизвестные, но интересные публикации.

Вы смотрите и ставите лайки

шаг 1

Алгоритм отслеживает это и подбирает контент

шаг 2

Вы видите интересные именно вам материалы

шаг 3

Интересные истории

В Дзене есть популярные медиа и талантливые блогеры. Ежедневно они создают тысячи историй на сотни разных тем. И каждый находит в Дзене что-нибудь для себя.

Примеры публикаций

В Дзене действительно много уникальных статей и видео. Вот несколько примеров популярного сейчас контента.

Дзен — простой, современный и удобный

Посмотрите на главные возможности сервиса и начните пользоваться всеми преимуществами Дзена.

Читайте о своих интересах.

Алгоритмы Дзена понимают, что вам нравится, и стараются показывать только то, что будет действительно интересно. Если источник вам не подходит — его можно исключить.

1/4

Тематические ленты.

С общей ленты со всеми статьями легко переключайтесь на тематические: кино, еда, политика, знаменитости.

2/4

Разнообразные форматы.

Открывайте разные форматы историй для чтения и общения. В приложении удобно читать статьи и смотреть видео, писать комментарии.

3/4

Оставайтесь в курсе событий!

Возвращайтесь к нужным статьям: добавляйте статьи в Сохранённое, чтобы прочитать их позже или сохранить в коллекции. Настройте уведомления, чтобы не пропустить самое интересное от любимых блогеров, медиа и каналов.

4/4

Читайте о своих интересах.

Алгоритмы Дзена понимают, что вам нравится, и стараются показывать только то, что будет действительно интересно. Если источник вам не подходит — его можно исключить.

1/4

Тематические ленты.

С общей ленты со всеми статьями легко переключайтесь на тематические: кино, еда, политика, знаменитости.

2/4

Разнообразные форматы.

Открывайте разные форматы историй для чтения и общения. В приложении удобно читать статьи и смотреть видео, писать комментарии.

3/4

Оставайтесь в курсе событий!

Возвращайтесь к нужным статьям: добавляйте статьи в Сохранённое, чтобы прочитать их позже или сохранить в коллекции. Настройте уведомления, чтобы не пропустить самое интересное от любимых блогеров, медиа и каналов.

4/4

Читайте о своих интересах.

Алгоритмы Дзена понимают, что вам нравится, и стараются показывать только то, что будет действительно интересно. Если источник вам не подходит — его можно исключить.

1/4

Тематические ленты.

С общей ленты со всеми статьями легко переключайтесь на тематические: кино, еда, политика, знаменитости.

2/4

Разнообразные форматы.

Открывайте разные форматы историй для чтения и общения. В приложении удобно читать статьи и смотреть видео, писать комментарии.

3/4

Оставайтесь в курсе событий!

Возвращайтесь к нужным статьям: добавляйте статьи в Сохранённое, чтобы прочитать их позже или сохранить в коллекции. Настройте уведомления, чтобы не пропустить самое интересное от любимых блогеров, медиа и каналов.

4/4

Дзен доступен во всем мире более чем на 50 языках

Смело рекомендуйте Дзен своим друзьям из других стран.

العَرَبِيَّة‎العَرَبِيَّة‎
Удобно пользоваться в смартфоне

У Дзена есть приложения для iOS и Android.

Пользуйтесь в браузере

Дзен доступен с любого устройства в вашем любимом браузере. Также Дзен встроен в Яндекс.Браузер.

Удобно пользоваться в смартфоне

У Дзена есть приложения для iOS и Android.

Пользуйтесь в браузере

Дзен доступен с любого устройства в вашем любимом браузере. Также Дзен встроен в Яндекс.Браузер.

Удобно пользоваться в смартфоне

У Дзена есть приложения для iOS и Android.

Пользуйтесь в браузере

Дзен доступен с любого устройства в вашем любимом браузере. Также Дзен встроен в Яндекс.Браузер.

© 2015–2021 ООО «Яндекс», 0+

Дизайн и разработка — Charmer

К сожалению, браузер, которым вы пользуйтесь, устарел и не позволяет корректно отображать сайт. Пожалуйста, установите любой из современных браузеров, например:

Яндекс.Браузер Google Chrome Firefox Safari

15 упражнений для тренировки мозга

Программистам без логики никуда. Поэтому время прокачать мозг: проверьте свои способности. Вам под силу эти логические задачи?

Полезно решать и логические задачи, и математические. Так вы развиваете логику и тренируете мозг. В силу профессии айтишнику крайне важно следить за тонусом своей главной «мышцы». Мозг любит задачки и головоломки, а ещё переключение внимания и отдых от рутины. Поэтому скорее приступим к развлечениям с пользой!

Логические задачи для разминки

1 задача

Поставьте правильное число вместо вопросительного знака:

4  5  6  7  8  9 
61 52 63 94 46 ?
Ответ

Числа нижнего ряда – квадраты чисел верхнего ряда с перестановкой цифр. Вместо знака ставьте число 18.

2 задача

Один парень в компании предложил друзьям такой спор:

– Спорим, я выставлю бутылку на середину комнаты и вползу в неё.

И получилось. Он победил.

Как парню посчастливилось это сделать?

Ответ

Он без труда вполз в неё – в комнату.

3 задача

Представьте ряд из шести чашек на столе. Три первые из них ничем не наполнены, а три следующие – с водой. Как добиться чередования пустых чашек и чашек с водой? Касаться разрешается только одной чашки. При этом толкать чашку чашкой запрещается.

Что вы предпримете?

Ответ

Возьмите пятую чашку, перелейте из неё воду во вторую и поставьте чашку на место.

4 задача

В санатории на лужайке двое мужчин заняты настольным теннисом. Один ударяет ракеткой так сильно, что теннисный шарик улетает далеко и попадает в трубу из стали. Труба зарыта в землю вертикально на три метра. Шарик лежит на дне трубы, то есть на расстоянии трёх метров от плоскости земли. У игроков нет другого шарика.

Ответьте, как спортсменам достать игральный шар без извлечения трёхметровой трубы из-под земли?

Ответ

Спортсмены наполнят трубу водой до краёв, и тогда шарик всплывёт.

5 задача

Получится ли у вас записать число 1000 с использованием только восьми восьмёрок и символов математического сложения?

Ответ
888 + 88 + 8 + 8 + 8 = 1000

Логические задачи основного комплекса

6 задача

Попробуйте установить принцип построения указанной последовательности:

8 2 9 0 1 5 7 3 4 6
Ответ

Последовательность цифр построена на основании алфавитного порядка их названий (восемь, два, девять, ноль и т. д.).

7 задача

Вообразите десятикилометровый мост через пролив. Максимальная нагрузка для него – 25 тонн. С начала этого моста стартовал грузовик, масса которого – ровно 25 тонн. Автомобиль продолжает движение к противоположному краю. Баланс моста пока не нарушен. Неожиданно, когда грузовик достиг середины этого путепровода, на него сел воробей со своим весом.

Вопрос к вам: приведёт ли вес птицы к нарушению балансировки и разрушению моста?

Ответ

Не приведёт. Так как грузовик преодолел путь до середины моста, что равно 5 км, расход потраченного топлива в разы превысил вес птицы.

8 задача

В одно и то же время к водному каналу приблизилось двое. Добраться до другого берега поможет лодка, которая рассчитана только на одну персону. Тем не менее, без стороннего участия оба переправились на противоположный берег на этой лодке.

Как люди справились?

Ответ

Люди приблизились к противоположным берегам водного канала.

9 задача

Отважного воина захватили слуги султана. Султан приказал отправить смельчака в темницу, где вместе с ним беспрерывно будут рядом два надзирателя. Один стражник исключительно правдив в разговоре, а второй лжёт. Внутри темницы оказались два закрытых входа. Войдёшь в один – выберешь «неволю навсегда», а в другой – «освобождение». Султан предложил воину избрать один вход: если это окажется «освобождение», то пленник свободен.

Воину позволили спросить надзирателей только один раз. То есть один вопрос на двоих надзирателей. Пленник не в курсе, кто из них лжец, а кто правдив. Надзиратели, несомненно, знают, какой вход подарит воину освобождение.

Какой вопрос задаст воин одному из стражников для достоверного определения входа, который означает «освобождение»?

Ответ

Воин задаст только один вопрос любому из надзирателей: «Если попросить твоего коллегу указать на вход «освобождение», то куда он направит меня?» В обеих ситуациях надзиратель укажет на «вход в неволю».

10 задача

В лесу десять родников с мёртвой водой: от первого до десятого. Мёртвая вода из родников с первого по девятый доступна каждому, а десятый родник во власти Кощея в пещере, в которую он никого не впускает. По вкусу и цвету мёртвую воду не отличить от обыкновенной, однако глоток воды из родника означает смерть. Спасёт только вода из родника с номером выше. Поэтому того, кто изначально выпьет десятую воду, ничего не спасёт.

Иванушка бросил вызов Кощею. Условились принести с собой чашку с водой и дать её осушить сопернику. Радости Кощея не было предела: «Ура! Я налью десятый яд, и Иванушка не спасётся! В то же время выпью то, что даст Иванушка, следом выпью десятый яд и останусь живым!»

В оговоренный день соперники столкнулись. Последовал честный обмен чашками. Содержимое выпито. При этом Кощей погиб, а Иванушка выжил.

Как Иванушка победил Кощея?

Ответ

Иванушка предложил Кощею обыкновенную воду. А Кощей, по неверным предположениям, «запил» её собственным десятым ядом. Перед встречей с соперником Иванушка выпил воду из любого родника. Вышло, что он запил яд Кощеевым десятым, что привело к нейтрализации яда.

Логические задачи для «растяжки»

11 задача

Отец решил задать своему сыну-школьнику каверзный вопрос: назови самое большое число. Ответ сына ошеломил отца, возразить было нечего.

Для программиста это дело лёгкое, правда? Но что сказал школьник?

Ответ

Тридцать первое. Предполагается число месяца.

12 задача

Воинственное племя захватило странника. Вождь хотел смерти страннику и позволил ему выбирать. Страннику разрешалось озвучить одну фразу. При правдивости фразы его сбросят с отвесной скалы. Окажись фраза лживая, и его отдадут львам на растерзание. Но странник подобрал такую фразу, которая подарила ему свободу.

Отгадайте, что это за фраза?

Ответ

Фраза: «Меня растерзают львы». Тогда, если бы вождь отдал странника львам на растерзание, то сказанная фраза стала бы правдивой, и его полагалось бы бросить с отвесной скалы. Но если странника сбросят со скалы, то фраза окажется лживой. Вождь посчитал, что исключительно правильным исходом будет подарить свободу страннику.

13 задача

После гулянки мужчина направлялся домой слегка навеселе. Он шёл по центру песчаной проселочной дороги. Путь не освещался лунным светом. К тому же, на дороге отсутствовали фонари. Одежда мужчины была чёрной. Вдруг на дорогу выехал автомобиль с выключенными фарами. В конце концов, водитель заметил мужчину и свернул.

Как ему удалось увидеть пешехода?

Ответ

На улице стоял день.

Логические задачи для заминки

14 задача

Вам даётся три письма. Одно придётся незамедлительно проглотить. В каждом письме найдёте пару предложений. Два предложения в одном письме истинные, в другом – ложные, а в третьем – пополам – истинное и ложное. Смотрите, какие там предложения:

Первое письмо:

  1. Не ешьте это письмо.
  2. Непременно съешьте второе письмо.

Второе письмо:

  1. Не стоит есть первое письмо.
  2. Жуйте третье письмо.

Третье письмо:

  1. Есть это письмо не стоит.
  2. Скорее съедайте первое письмо.

Поделитесь своим мнением, какое письмо съесть?

Ответ

Третье письмо.

15 задача

Вы очутились в помещении, в котором четыре двери и крошечное окошко. Три двери фальшивые, то есть за ними сразу кирпичная кладка. И одна дверь с выходом на улицу. Вам дали ключ, который открывает все четыре двери, однако вы без понятия, какая дверь выведет на улицу. Попытаться можно один раз. При открывании одной двери оставшиеся замки блокируются механически и безвозвратно. Вдобавок комната тёмная и слегка озаряется светом одной свечи.

Какие вы примете меры, чтобы отыскать единственную дверь, которая ведёт на улицу?

Ответ

Стоит распахнуть окошко и подставлять свечу по очереди к дверям: к щелям или к замочной скважине. При этом внимательно смотреть на пламя свечи. Колебание пламени будет указывать на выход.

Эти логические задачи показались вам лёгкими или не очень?

Попробуйте другие логические задачи:

8 сложных вопросов на интервью от Google, Apple и Facebook

Технологические компании Кремниевой долины известны тем, что задают довольно сумасшедшие вопросы на собеседовании…

Я хотел выяснить, что именно включают эти вопросы. И как им сложно ответить.

Итак, я провел день на Glassdoor.com и нескольких других сайтах, чтобы придумать 8 самых сложных и самых интересных логических вопросов на собеседовании.

И не только от каких-либо компаний … Мы собираемся взглянуть на 4 технологических гиганта, известных своими сложнейшими собеседованиями:

  1. Facebook
  2. Google
  3. Apple
  4. LinkedIn

Пора посмотреть, сколько вы сможете ответить!

Facebook Brain Teaser Интервью Вопросы и ответы:

Вопрос 1:

Русский гангстер похищает вас. Он последовательно вставляет две пули в пустой шестизарядный револьвер, раскручивает его, направляет вам в голову и стреляет. * клик * Ты еще жив. Затем он спрашивает вас: «Вы хотите, чтобы я снова повернул его и выстрелил или сразу же снова нажал на курок?» Какова вероятность того, что вас застрелят, для каждого варианта?

Ответ…

Ключевой намек здесь в том, что пули были заряжены рядом друг с другом.

Есть 4 способа расположить револьвер последовательными пулями так, чтобы первый выстрел был холостым.Это возможные сценарии:

  1. (xBBxxx)
  2. (xxBBxx)
  3. (xxxBBx)
  4. (xxxxBB)

В двух других сценариях вас застрелили с первой попытки. (BBxxxx) или (BxxxxB)

Теперь посмотрите на второй слот в этих 4 возможных сценариях выше. Ваши шансы получить выстрел — 1/4 или 25%. (Только №1 может вас застрелить)

Но если вы вернетесь… осталось 2 пули и всего 6 слотов. 2/6 или 33%.

Вопрос 2:

Вы собираетесь сесть на самолет в Сиэтл. Вы хотите знать , идет ли дождь. Вы звоните трем случайным друзьям, которые там живут, и спрашиваете каждого, идет ли дождь. У каждого друга есть 2/3 шанса сказать вам правду и 1/3 шанса обмануть вас ложью. Все 3 друга говорят вам, что «да», идет дождь. Какова вероятность того, что в Сиэтле действительно идет дождь?

Ответ…

Для того, чтобы в Сиэтле пошел дождь, достаточно, чтобы один из ваших друзей сказал правду.

Быстрее всего просто рассчитать шансы, что все трое лгут и что дождь не идет.

У каждого друга 1/3 шанс солгать. Умножьте шансы вместе … вы получите 1/27 (1/3 * 1/3 * 1/3).

Но мы еще не закончили… 1/27 — вероятность того, что все 3 друга солгали одновременно.

Вероятность того, что хотя бы 1 человек сказал вам правду? 26/27 или около 96%, что в Сиэтле идет дождь.

Google Brain Teaser Интервью Вопросы и ответы:

Вопрос 3:

У вас есть кувшин на 3 галлона и кувшин на 5 галлонов, как вы точно отмеряете 4 галлона?

Ответ…

Мы знаем, что не можем получить окончательный результат в 3-галлонном кувшине. Это переполнится. У нас должно получиться 4 галлона в 5-галлонном кувшине.

Сначала наполните кувшин на 3 галлона.

Затем вылейте 3 галлона в кувшин на 5 галлонов.

Теперь кувшин на 3 галлона пуст, а кувшин на 5 галлонов содержит 3 галлона.

Снова наполните кувшин объемом 3 галлона. Медленно перелейте в кувшин объемом 5 галлонов. Подойдет только 2 галлона, потому что в нем уже есть 3. Теперь он заполнен.

В кувшине емкостью 3 галлона остается ровно 1 галлон.

Выгрузите кувшин на 5 галлонов.

Налейте 1 галлон в кувшин на 5 галлонов.

Наполните кувшин на 3 галлона еще раз и вылейте его в кувшин на 5 галлонов! У вас ровно 4 галлона (и, возможно, работа в Google)

Вопрос 4:

Почему крышки люков круглые?

Ответ…

Хорошие новости: если вы устали от вопросов по математике, этот вопрос даст вам передышку.

Крышки люков круглые, потому что это единственная форма, которая не может провалиться сквозь себя. Крышка никогда не может случайно упасть в отверстие.

Microsoft, как известно, задает этот вопрос, и, согласно Glassdoor.com, Google тоже задает этот вопрос.

Apple Brain Teaser Интервью Вопросы и ответы:

Вопрос 5:

Есть три коробки: одна содержит только яблоки, одна содержит только апельсины, а одна — яблоки и апельсины. Ящики были неправильно промаркированы, поэтому фактическое содержимое ящика не идентифицируется ни на одной этикетке.Открывая всего одну коробку, вы, не заглядывая в нее, достаете один фрукт. Глядя на фрукты, как сразу правильно промаркировать все коробки?

Ответ…

Итак, вы знаете, что все 3 поля имеют неправильную маркировку.

Перейдите в коробку с надписью «Яблоки + апельсины». Поскольку этикетка неправильная, на ней должно быть одно или другое.

Это коробка, из которой можно взять один фрукт. Что бы ни вышло, это то, что находится в этой коробке. Если вы достали яблоко, в коробке только яблоки. Если вы вынули апельсин, то наоборот.

Вот здесь немного хитрости. Но мы почти закончили…

Допустим, вы схватили яблоко. Переместите метку «Яблоки» в это поле. Теперь он правильно помечен.

Вы знаете, что поле «Апельсины» по-прежнему неправильно помечено (потому что все три были неправильно помечены, и вы не прикасались к нему). И вы знаете, что это не «Яблоки».

Значит, это должно быть «Яблоки + Апельсины».

Последний ящик — «Апельсины».

Тот же процесс, описанный выше, будет работать, если вы вытащили апельсин в начале.

Вопрос 6:

У вас, , есть 100 монет, лежащих на столе, каждая из которых имеет верхнюю и заднюю стороны . 10 из них — решка, 90 — решка. Вы не можете ни пощупать, ни увидеть, ни каким-либо иным образом узнать, какие 10 игроков хедз-ап. Ваша цель: разделить монеты на две стопки так, чтобы в каждой стопке было одинаковое количество хедз-ап монет.

Ответ…

По чистой случайности… это трюк, который мой друг Майк показал мне прошлым летом.Тогда это поразило меня, но, надеюсь, будет иметь смысл, когда я буду писать.

Вы хотите, чтобы в каждой стопке было одинаковое количество голов. Сейчас их 10 штук. Вы не знаете какой, но это не имеет значения. Все, что вам нужно сделать … взять любые 10 монет из 100, сложить их в отдельную стопку и перевернуть эти 10.

Это стопка №1.

Кучка № 2 — это оставшиеся 90 монет, не поднятые. Просто оставь их.

Готово. Серьезно.

Это можно сделать с любым количеством монет.Если бы у вас было 20 монет, а 18 были орлами, вам нужно было бы взять 18 из них (неважно какие) в отдельную кучу и перевернуть те 18. Это стопка №1.

Если у вас было 10 монет и 3 орла, вы бы взяли 3 случайные монеты в новую стопку и перевернули эти 3 в первую стопку, а остальные — во вторую стопку.

Сумасшедший, правда?

Если вы мне не верите, просто возьмите несколько пенсов и попробуйте.

Нет никаких исключений, и для начала не обязательно должно быть четное количество «голов».Также может быть нулевым. Или все.

LinkedIn Brain Teaser Interview Вопросы и ответы:

Вопрос 7:

Вы находитесь в комнате с тремя выключателями, каждый из которых c управляет одной из трех лампочек в соседней комнате. Вам необходимо определить, какой переключатель какой лампочкой управляет. Для начала все огни выключены, и вы не можете видеть одну комнату из другой. Вы можете осмотреть другую комнату только один раз. Как узнать, какие переключатели к каким лампочкам подключены?

Ответ…

Назовем переключатели 1, 2 и 3.

Оставьте переключатель 1 выключенным.

Включите переключатель 2 на десять минут.

Теперь выключите его и быстро включите переключатель 3.

Зайдите в комнату и осмотрите…

Лампочка, которая все еще теплая, но не горит, управляется переключателем 2. В настоящее время горит переключатель 3. Последний — переключатель 1.

Вопрос 8:

Сколько мячей для гольфа поместится в Боинг 747?

Ответ…

Последний вопрос сложен, но они не ожидают, что вы получите точный ответ.Если вам задают такой вопрос (а есть масса вариантов — баскетбольные мячи в комнате, мобильные телефоны на Манхэттене и т. Д.), Они захотят увидеть ваш мыслительный процесс.

Менеджер по найму будет смотреть, как вы работаете с этим, и попытается это понять.

Если вы можете разбить проблему на более мелкие части, сохранять спокойствие и получить ответ, который не идеален, но достаточно близок, вы отлично справились.

Они могут даже не знать ответа. Они просто хотят увидеть, как вы подходите к тому, что очень сложно.

Примечание. Прочтите эту статью, чтобы узнать о 5 других важных вещах, которые хочет видеть каждый менеджер по найму.

На заметку, что вы можете извлечь из этого?

В вопросе 8 выше подчеркивается довольно хороший момент, который следует помнить во время собеседований . ..

На собеседовании есть много вопросов, в которых менеджер по найму ценит ваш мыслительный процесс … иногда даже больше, чем правильный ответ.

Так что, если вы в тупике, поговорите немного вслух и объясните, о чем вы думаете.Задайте вопрос, если нужно. Попробуйте разбить его на более мелкие части.

Конкретным знаниям можно научить, но они не могут научить вас решать проблемы. Вот почему они задают логичные вопросы на собеседовании, и почему они задают вопросы, в которых, как они ожидают, вы будете бороться или не уверены.

Если вы слышите подобные вопросы, это не значит, что у вас все плохо. Просто сохраняйте спокойствие, проведите их через свой мыслительный процесс, покажите, что вы применяете логический подход, и у вас будет отличный шанс получить работу (даже если в конце концов вы не дадите идеального ответа!)

I Я много говорю о том, как это сделать и почему это важно, в моем руководстве по ответам на собеседование.

Вас могут устроить на работу, даже если вы дадите «неправильные» ответы на эти вопросы.

Вот небольшая история: у меня степень в области финансов, а это значит, что я тоже хорошо изучил бухгалтерский учет. В начале моей карьеры у меня было телефонное собеседование на должность бухгалтера.

Короче говоря, я не мог ответить даже на самые простые бухгалтерские вопросы. Действительно простые вещи, которые вы изучаете на первом курсе колледжа.

Почему?

Прошло слишком много времени с тех пор, как я учился в колледже, и я забыл даже основы.И я явно плохо подготовился к интервью!

Но я попытался споткнуться и вспомнить, что мог, говоря о том, о чем я думал. Сказать что-то вроде «ну, это не может быть правильным, потому что ___. Так что это должно быть связано с ___ ».

Я добился определенных успехов. Но я определенно не пришел к правильному ответу, даже после трех минут ходьбы я и интервьюер вслух.

Но меня все равно пригласили на следующий раунд собеседования (полный день, собеседование на месте).

Почему? Потому что менеджеру по найму понравился мой подход к решению проблемы, которую я не сразу знал, как решить. Вот почему быть прозрачным и продемонстрировать свой мыслительный процесс — один из советов для собеседований, который я буду повторять снова и снова.

И это самый важный вывод, который, я надеюсь, вы получите, прочитав эти вопросы-головоломки выше (вместе с развлечениями).

Вы можете сделать то же самое, что и я, и получить больше предложений о работе … даже если вы дадите несколько неправильных ответов на такие сложные вопросы!

Итог: Не паникуйте, когда вам задают вопрос, которого вы не знаете; используйте это как возможность показать, как именно вы работаете с вещами.Будьте с ней уверены, расслабьтесь, улыбнитесь. Помните … вы даете менеджеру по найму то, что он хочет!

ОБНОВЛЕНИЕ:

Если вам предстоит собеседование, и вы не хотите ничего оставлять на волю случая, я создал новое руководство, в котором вы можете скопировать мой точный пошаговый метод получения предложений о работе. . Вы можете получить более подробную информацию здесь.

Понравился пост? Прикрепите его, чтобы сохранить в следующий раз!

Tricky Logical Reasoning Questions + Answers

Некоторое время назад мы написали рассказ о некоторых из лучших головоломок.Теперь это не такие уж случайные вопросы, это актуальные вопросы с логическим обоснованием, размещенные в Интернете учениками старших классов. Но, как назло, они достаточно жесткие, чтобы вызвать раздражение даже у взрослых. Хотя некоторые из них могут быть довольно простыми, вам, возможно, придется поразмыслить над другими в течение нескольких минут. Сделайте нам одолжение и попробуйте решить их, прежде чем щелкнуть, чтобы открыть. Как сказал наш премьер-министр: «Да пребудет с вами сила».

Сложные вопросы логического рассуждения с ответами

12 вопросов логического рассуждения, которые мгновенно поставят вас в тупик:

1.Есть комната без дверей и без окон. Найден мужчина подвешенным к потолку. На полу лужа воды.

Как он умер?

2. Есть три дома. Один красный, один синий и один белый. Если красный дом находится слева от дома посередине, а синий — справа от дома посередине, то где же белый дом?

3. Мужчина заходит в ресторан, официант здоровается, адмирал. Почему официант назвал человека адмиралом?

4.Позавчера Крису было 7 лет. В следующем году ему исполнится 10 лет. Как такое возможно?

5. Есть один человек, убивший свою мать. Он родился раньше своего отца и женился на более чем 100 женщинах, ни с кем не разводясь. Тем не менее, все его знакомые считали его нормальным. Почему?

Также прочтите: Вопросы на собеседовании IAS на хинди

6. Две лошади родились одновременно. Они оба путешествовали по миру и умерли одновременно. Однако они не дожили до одного возраста.Как?

7. Мальчик и врач ловили рыбу. Мальчик — сын доктора, но доктор не отец мальчика. Кто доктор?

8. Найдите лишнего —


ПЕРВЫЙ, ВТОРОЙ, ТРЕТИЙ, ЧЕТВЕРТОЙ, ПЯТЫЙ, ШЕСТОЙ, СЕДЬМОЙ, ВОСЬМОЙ

9.

Переставьте следующие буквы в одно слово: НОВАЯ ДВЕРЬ.

10. Стоит ли старая купюра в сто рупий дороже новой?

11. Есть две сестры: одна рожает вторую, а она, в свою очередь, рожает первую.Кто эти две сестры?

12. На какой единственный вопрос вы не можете ответить утвердительно?

сложных математических головоломок с ответами — сложные вопросы-головоломки

Вопросы-головоломки с логическими рассуждениями часто задают в банковских PO, SSC, государственных экзаменах и многих вступительных тестах. В основном это головоломки, основанные на аранжировке или картинках. Такие вопросы предназначены для проверки ваших аналитических, дедуктивных способностей и способностей к решению проблем. Вот сборник различных логических головоломок с ответами.Этот набор содержит 10 сложных головоломок, основанных на принципах логического мышления.

Решите данные вопросы и проверьте свой уровень подготовки:

Q.1. Переместите 4 спички, чтобы образовать 3 квадрата разного размера.

Решение и объяснение

Это изображение после перемещения 4 спичек:

Q.2. На скамейке сидят мужчина и женщина.

«Я мужчина», — говорит человек с каштановыми волосами.

«Я женщина», — говорит человек с черными волосами.

Если хотя бы один из них лжет, кто мужчина, а кто женщина?

Решение и объяснение

Простой анализ показывает, что люди либо оба лгут, либо оба говорят правду. Поскольку нам говорят, что по крайней мере один из них лжет, человек с каштановыми волосами — это женщина, а человек с черными волосами — это мужчина.

(По крайней мере, один означает один или несколько)

Q.3. Лео, Долли и Томми связаны друг с другом.

и. Среди этих троих — законная супруга Лео, родная сестра Долли и невестка Томми.

ii. Законный супруг Лео и брат Долли одного пола.

Кто вы знаете женатого мужчину?

Решение и объяснение

Если супруга Лео — Долли, то родным братом Долли не может быть Лео, а должен быть Томми; тогда невестка Томми не может быть Долли, а должна быть Лео. Если супруга Лео — Томми, то невестка Томми не может быть Лео, а должна быть Долли; тогда родная сестра Долли не может быть Лео, а должна быть Долли; тогда брат и сестра Долли не могут быть Томми, а должны быть Лео.Потом, в любом случае, все трое Лео, Долли. И Томми числится, и невестка Томми — женщина. Итак, из [2], супруга Лео и брат Долли оба мужчины.

Итого:

Супруга Лео, сестра Долли, сестра Томми

Мужской Мужской Женский

Чехол I Dolly Tommy Leo

Чехол II Tommy Leo Dolly

Случай II исключается, поскольку Лео и Томми не могут быть мужчинами и женаты друг на друге.Итак, Случай I — правильный, и вы знаете, что Долли женатый мужчина. Лео — замужняя женщина, Долли и Томми — братья, а Лео — невестка Томми.

Q.4. Президент решает дать заключенному, приговоренному к смертной казни, последний шанс выжить. Есть 2 двери, одна — дверь жизни, а другая — дверь смерти. У каждой двери стоит один охранник, который знает о дверях. Однако один из них всегда говорит правду, а другой всегда лжет.Невозможно определить, какая дверь является дверью жизни, а какая — дверью смерти. Невозможно определить, кто из сотрудников службы безопасности говорит правду. Заключенный может задать только один вопрос любому из охранников. После этого ему нужно выбрать дверь. Если он войдет в дверь смерти, то будет казнен. Если он войдет в дверь жизни, он получит жизнь. Он действительно выбрал дверь жизни и жил. Какой вопрос он задал? Как он выбрал дверь после того, как получил ответ от одного из охранников?

Решение и объяснение

Предположим, что охранник X находится у двери X, а другой человек — у двери Y.Пусть заключенный подойдет к человеку X и задаст этот вопрос: «Если я спрошу человека Y, какая дверь является дверью Y, то что он ответит: дверь жизни или дверь смерти?»

Если ответ — дверь жизни (дверь X), он может войти в дверь Y. Если ответ — дверь смерти (дверь Y), он может выбрать дверь Y и войти. Это потому, что вопрос проходит через 2 уровня вопросов от 2 человек. Общий ответ всегда будет ложью, потому что всегда один человек лжет, а другой говорит правду.

Q.5. Буквы от D до Z делятся на 4 группы. Предположим, что мы использовали шрифт Arial. К какой группе принадлежит буква G? К какой группе принадлежит буква, к которой я принадлежу?

1 К j
2 D E
3 u В M
4 N S O
Решение и объяснение

Все буквы в первом ряду не симметричны.Все буквы второго ряда симметричны сверху и снизу. Все буквы в третьем ряду симметричны слева и справа. Все буквы в последнем ряду полностью симметричны. Следовательно, G должна перейти в первую строку. Я должен перейти к последнему ряду.

Q.6. По дороге в Тунгнат Вики достигла развилки. Он мог пойти любым из двух путей. Но только один из них ведет в город. Но, к счастью, рядом стояли двое мужчин, однако один из них всегда лжет, а другой всегда говорит правду и неизвестно, кто есть кто.Поскольку мужчины не очень-то любят помогать, разрешается задать одному из них только один вопрос. Какой вопрос ему задать?

Решение и объяснение

Спросите одного из мужчин: «Могу ли я спросить человека, стоящего рядом с вами: какой путь в город?»,

что бы он ответил? »

Если он спросит об этом лжеца, он укажет ему неверный путь.

Если он спросит об этом того, кто говорит правду, он также укажет ему неверный путь. Поэтому, задав вопрос, выберите другой путь.Это приведет вас в город.

Q.7. В поезде S, R и J — это пожарный, тормозной мастер и инженер, но могут НЕ быть в порядке. Также в поезде находятся три бизнесмена с одинаковыми именами: мистер С., мистер Р. и мистер Дж. Используя приведенные ниже подсказки, можете ли вы определить личность инженера?

  1. Г-н Р. живет в Дехрадуне.
  2. Тормозщик живет ровно на полпути между Нойдой и Дехрадун.
  3. Мистер J зарабатывает ровно рупий. 4,00,000 в год.
  4. Ближайший сосед тормозящего, один из пассажиров, зарабатывает ровно в три раза больше, чем тормозящий.
  5. S превосходит пожарного в бадминтоне.
  6. Пассажир, имя которого совпадает с именем тормозного мастера, живет в Нойде.
Решение и объяснение

a) Мистер Р. живет в Дехрадуне, и ближайший сосед тормозящего зарабатывает ровно в 3 раза больше, чем тормозящий.

Следовательно, ни г-н R, ни г-н J не являются ближайшим соседом тормозящего, поэтому это должен быть г-н.С.

б) S побеждает пожарного в бадминтоне и пассажира, имя которого совпадает с именем тормозного мастера в Нойде. Г-н Р. живет в Дехрадуне, а г-н С. живет между Нойдой и Дехрадун. Следовательно, это должен быть мистер J, который живет в Нойде, а J — тормозной мастер.

c) S не тормозник и не пожарный. Он, должно быть, инженер.

Имя Город Род занятий Заработок
S Нойда Инженер
R Дехрадун Пожарный
Дж Ч / б Нойда и Дехрадун Тормозной механизм
Mr.S Ч / б Нойда и Дехрадун
г-н J Нойда 4,00,000
Г-н Р Дехрадун

Q.8. Четыре чашки перевернуты на прилавок. В каждой чашке одинаковое количество конфет и указание количества конфет в ней. Заявления следующие: пять или шесть, семь или восемь, шесть или семь, семь или пять.Только одна декларация верна. Сколько конфет под каждой чашкой?

Решение и объяснение

Поскольку указано, что только одно из четырех объявлений является правильным, правильный номер не может появляться более чем в одном объявлении. Если он присутствует более чем в одном объявлении, то более чем одно объявление будет правильным. Следовательно, под каждой чашкой находится 8 конфет.

Q.9. У вас 14 яблок. Ваш друг Моника забирает 3 и дает вам 2. Вы бросаете 7, но берете 4.Бхим берет 4 и дает 5. Вы берете одну у Моники и отдаете ее Бхиму в обмен на еще 3. Вы отдаете эти 3 Монике, и она дает вам яблоко и апельсин. Приходит Физа, берет яблоко, которое дала вам Моника, и дает грушу. Вы даете грушу Бхиму в обмен на яблоко. Затем Физа берет яблоко у Моники, дает Бхиму вместо апельсина и дает апельсин вместо яблока. Сколько у вас груш?

Решение и объяснение

Нет. Физа дала вам грушу в обмен на яблоко, которое подарила вам Моника.И ты отдал эту грушу Бхиму в обмен на яблоко. На всех остальных биржах были яблоки и / или апельсины.

Q.10. Пять друзей с фамилиями Панвала, Леттервала, Талавала, Чунавала и Радхивала имеют следующие имя и отчество.

Четыре из них имеют имя и отчество Верма.

Трое из них имеют имя и отчество Канта.

Два из них имеют имя и отчество Раджеш.

У одного из них есть имя и отчество Киаш.

Леттервала и Талавала, либо оба названы Канта, либо ни один не назван Канта. И Панвала, и Леттервала зовут Раджеш, или Талавала, и Чунавела — Раджеш. Чунавала и Радхивала не оба названы Верма. Кого зовут Киаш?

Решение и объяснение

Из (1) и (7) ясно, что Панвала, Леттервала и Талавала называются Верма.

Из (6) и (5), если Леттервала или Талавала оба названы Канта, то у любого из них будет три имени i.е. Верма, Канта и Раджеш. Следовательно, Леттервала и Талавала не названы Канта. Это означает, что Панвала, Чунавала и Радхивала названы Кантой. Теперь ясно, что Талавала и Чунавела зовут Раджеш. Кроме того, Леттервала зовут Киаш.

загадок для взрослых | Лучшие головоломки и логические головоломки



Приказы короля создают адскую головоломку

(Сложность: легкая)

Король Нупе из королевства Катан настолько обожает своих двух дочерей, что решает королевство было бы лучше иметь больше девочек, чем мальчиков, и он издает следующий указ: Все детородные пары должны продолжать рожать детей, пока у них не родится дочь!

Но, чтобы избежать перенаселения, он издает дополнительный указ: все детородные пары прекратят заводить детей, как только у них родится дочь! Его подданные немедленно начинают выполнять его приказы.

Каково ожидаемое соотношение девочек и мальчиков в Катане по прошествии многих лет?

Подсказка

Вероятность того, что каждый ребенок родится девочкой, составляет, конечно, 50 процентов.

Решение

Готовы к решению? Нажмите здесь, чтобы узнать, правы ли вы.



Сколько яиц несёт эта курица?

(Сложность: Легкая)

Эта задача в честь моего отца, Гарольда Фейвсона.Благодаря ему я люблю решать математические головоломки, и это одна из первых задач (из многих), которые он дал мне, когда я рос.

Половина курицы откладывает полтора яйца за полтора дня. Сколько яиц откладывает одна курица за день?

Решение

Готовы к решению? Щелкните здесь, чтобы убедиться, что вы правы .


Математическая задача с золотой цепью обманчиво проста

(Сложность: средняя)

Вы роетесь на чердаке своей прабабушки и находите пять коротких цепочек, каждая из которых состоит из четырех золотых звеньев. Вам приходит в голову, что если вы объедините их все в одну большую петлю из 20 звеньев, у вас получится невероятное ожерелье. Итак, вы приносите его ювелиру, который говорит вам, что стоимость изготовления ожерелья будет составлять 10 долларов за каждую золотую связь, которую она должна разорвать, а затем снова запечатать.

Сколько это будет стоить?

Решение

Готовы к решению? Нажмите здесь, чтобы узнать, правы ли вы.


Попробуйте решить эту головоломку с пиклболом

(Сложность: «ЖЕСТКИЙ»)

Кенни, Эбби и Нед собрались на круговой турнир по пиклболу, где, как обычно, Победитель остается после каждой игры, чтобы сыграть с тем, кто выбыл из этой игры.В конце их пиклбольного дня Эбби устала, сыграв последние семь игр подряд. Кенни, который менее запутан, подсчитывает количество сыгранных игр:

Кенни сыграл восемь игр

Эбби сыграл 12 игр

Нед сыграл 14 игр

Кто кого выиграл в четвертой игре?

Подсказка

Сколько всего игр было сыграно?

Решение

Готовы к решению? Щелкните здесь, чтобы убедиться, что вы правы .


Наша загадка с автоматическим выключателем — чистое зло. Извини.

(Сложность: 🚨HARD 🚨 )

Коробка выключателя в вашем новом доме находится в неудобном углу вашего подвала. К своему огорчению, вы обнаруживаете, что ни один из 100 автоматических выключателей не имеет маркировки, и сталкиваетесь с устрашающей перспективой сопоставления каждого автоматического выключателя с соответствующим светом. (Предположим, что каждому автоматическому выключателю соответствует только одна лампа.)

Для начала вы включаете все 100 источников света в доме, а затем спускаетесь в подвал, чтобы начать обременительный процесс картирования. При каждом посещении подвала вы можете включать и выключать любое количество автоматических выключателей. Затем вы можете бродить по коридорам своего дома, чтобы узнать, какие огни включены, а какие нет.

Какое минимальное количество поездок в подвал необходимо для привязки каждого автоматического выключателя к каждому свету?

Подсказка

Решение , а не включает в себя включение или выключение выключателей света в вашем доме или определение температуры лампочек. Возможно, вы захотите сначала попробовать решение для случая с 10 автоматическими выключателями без маркировки.

Решение

Готовы к решению? Щелкните здесь, чтобы убедиться, что вы правы .


Два поезда. Две бабушки. Сможете ли вы решить эту сложную математическую загадку?

(Сложность: средняя)

Две бабушки Джесси хотят видеть его каждые выходные, но они живут на противоположных концах города. В качестве компромисса он говорит им, что каждое воскресенье он будет идти к ближайшей к его квартире станции метро в произвольное время дня и сесть на следующий прибывший поезд.

Если это поезд, идущий на север, он посетит свою бабушку Эрику в верхней части города, а если это окажется поезд, едущий на юг, он посетит свою бабушку Кара в центре города. Обе его бабушки согласны с этим планом, так как они знают, что поезда и на север, и на юг ходят каждые 20 минут.

Но через несколько месяцев бабушка Кара жалуется, что видит его только в одно из пяти воскресений. Джесси обещает, что он действительно направляется на станцию ​​каждый день в случайное время. Как такое может быть?

Подсказка

Поезда всегда прибывают в назначенное время.

Решение

Готовы к решению? Щелкните здесь, чтобы убедиться, что вы правы .


Вот действительно чертовски сложная математическая задача о муравьях

(Сложность: «ТРУДНО»)

Макс и Роуз — братья и сестры муравьев. Они любят соревноваться друг с другом, но всегда равны, поскольку на самом деле ползают с одинаковой скоростью. Поэтому они решают организовать гонку, в которой один из них (надеюсь) победит.

В этой гонке каждый из них стартует в нижнем углу кубоида, а затем ползет так быстро, как только может, чтобы добраться до крошки в противоположном углу. Размеры их кубов, как показано на рисунке:

.

Лаура Фейвесон

Если они оба выберут самый короткий путь, чтобы добраться до своей крошки, , кто первым доберется до своей крошки? (Не забывайте, что это муравьи, поэтому, конечно, они могут лазить по краям или поверхности кубоида. )

Подсказка

Помните: мыслите нестандартно.

Решение

Готовы к решению? Нажмите здесь, чтобы убедиться, что вы правы .


Эта загадка с мятой и мятой практически невозможна

(Сложность: «СЛОЖНО»)

Вы встречаетесь со своей подругой Кэрин в «конфетке», которая работает следующим образом: есть куча из 100 карамелей и одной котлеты из мяты. Вы с Кэрин будете ходить взад и вперед, беря по крайней мере одну и не более пяти карамель из стопки конфет за каждый ход.Тот, кто уберет последнюю карамель, также получит пирожок с мятой. А вы любите пирожков с мятой.

Предположим, Кэрин позволяет вам решать, кто пойдет первым. Кого выбрать, чтобы выиграть пирожок с мятой?

Подсказка

Сначала найдите стопку из 10 карамелей.

Решение

Готовы к решению? Нажмите здесь, чтобы убедиться, что вы правы .


Сможете ли вы разгадать великую американскую загадку о железнодорожных путях?

(Сложность: Средняя)

Эту задачу предложил физик П.Джеффри Ангар.

Наконец, Great American Rail-Trail через всю страну завершен! Идите вперед, похлопайте себя по спине — вы только что установили самый длинный поручень в истории мира, протяженностью 4000 миль от начала до конца. Но сразу после церемонии открытия ваш помощник напоминает вам, что металл, который вы использовали для поручня, летом немного расширяется, так что его длина в общей сложности увеличивается на один дюйм.

«Ха!» вы говорите: «Один дюйм в поручне длиной 4000 миль? Ничего подобного! » Но… ты прав?

Предположим, что когда поручень расширяется, он выгибается вверх в самом слабом месте, которое находится в центре.Насколько выше пешеходы в центре страны должны будут подниматься летом, чтобы ухватиться за поручни? То есть на рисунке ниже что такое h ? (Для целей этого вопроса игнорируйте кривизну Земли и предполагайте, что след представляет собой прямую линию. )

Лаура Фейвесон

Подсказка

Пифагор — увлекательная историческая личность.

Решение

Готовы к решению? Нажмите здесь, чтобы узнать, правы ли вы.


Эта загадка похожа на особо жестокую задачу SAT. Сможете ли вы найти ответ?

(Сложность: средняя)

Аманда живет со своим сыном-подростком Мэттом в сельской местности — на машине от школы Мэтта. Каждый день Аманда в одно и то же время покидает дом, едет в школу с постоянной скоростью, забирает Мэтта именно тогда, когда его шахматный клуб заканчивается в 17:00, а затем они немедленно возвращаются домой с той же постоянной скоростью.Но однажды Мэтт плохо себя чувствует, поэтому он рано оставляет шахматную тренировку и отправляется домой на своем портативном самокате.

После того, как Мэтт бежит в течение часа, Аманда встречает его в своей машине (по своему обычному маршруту, чтобы забрать его), и они возвращаются вместе, прибыв домой на 40 минут раньше, чем обычно. Сколько шахматных тренировок пропустил Мэтт?

Подсказка

Рассмотрим случай, когда Аманда встречает Мэтта именно тогда, когда она выходит из их дома.

Решение

Готовы к решению? Щелкните здесь, чтобы убедиться, что вы правы .


Сможете ли вы получить этих 3 кинозвезд за рекой?

(Сложность: средняя)

Три кинозвезды, Хлоя, Лекса и Джон, снимают фильм в Амазонке. Они очень известны и требуют особого ухода, поэтому их агенты всегда с ними. Однажды, после съемок сцены глубоко в тропическом лесу, трое актеров и их агенты решают вернуться на базу пешком. Вдруг они подходят к большой реке.

На берегу реки они находят небольшую гребную лодку, но она достаточно велика, чтобы вместить две лодки одновременно.Уловка? Никому из агентов не нравится оставлять свою кинозвезду с другими агентами, если их тоже нет. Они не верят, что другие агенты не попытаются переманить их звезду.

Например, агент Хлои в порядке, если Хлоя и Лекса одни в лодке или на одном из берегов реки, но определенно не , если агент Лексы тоже с ними. Так как же им всем перебраться через реку?

Подсказка

Есть не один способ решить эту проблему.

Решение

Готовы к решению? Щелкните здесь, чтобы убедиться, что вы правы .


Эта до смешного сложная загадка — наша дань уважения позднему гению математики. Вы можете это понять?

(Сложность: 🚨HARD 🚨)

11 апреля Джон Хортон Конвей , блестящий математик, который страстно и весело любил головоломки и игры, умер от осложнений, вызванных COVID-19.Конвей — изобретатель одной из моих любимых легендарных задач (не для слабонервных) и, как известно, Game of Life . Я создал эту задачу в его честь.

Кэрол создавала генеалогическое древо, но у нее возникли проблемы с отслеживанием даты рождения своей матери. Единственной уликой, которую она нашла, было письмо, написанное ее дедушкой бабушке в день рождения ее матери. К сожалению, некоторые символы были размыты, представленные здесь «___» .(Длина линии не отражает количество нечетких символов.)

«Дорогая Вирджиния,

» Мало ли, когда я направлялся на работу в понедельник утром, что к вечеру у нас родится красивая девочка. И в годовщину нашей свадьбы, не меньше! Это заставляет меня вспоминать тот невероятный день выходных, 27-е, J___, 19___ , когда мы впервые поделились нашей клятвой создать вместе семью, и, ну, вот и мы! С восьмилетним юбилеем, любовь моя.

С любовью, Эдвин »

Вопрос: когда родилась мать Кэрол?

Подсказка

Эта задача основана на правиле Судного дня Конвея.

Решение

Готовы к решению? Щелкните здесь, чтобы убедиться, что вы правы .


Чтобы решить эту запутанную математическую загадку, вам нужен только один пояс и одна земля

(Сложность: средняя)

Представьте, что у вас очень длинный пояс. Что ж, очень долго, правда… на самом деле, этого достаточно, чтобы он мог плотно обернуться вокруг всей нашей планеты. (Для простоты предположим, что Земля идеально круглая, без гор, океанов или других преград на пути пояса.)

Вы, конечно, очень гордитесь своим поясом. Но тут появляется ваш брат Питер — и, к вашему недовольству, он производит ремень , всего на немного длиннее вашего. Он хвастается, что его пояс длиннее ровно на его рост: 6 футов.

Если бы Питер также обернул свой пояс по окружности Земли, как далеко над поверхностью он смог бы подвесить пояс, если бы он натянул его туго и равномерно?

Подсказка

Окружность Земли составляет около 25 000 миль или 130 миллионов футов … но вам не нужно знать это, чтобы решить эту проблему.

Решение

Готовы к решению? Щелкните здесь, чтобы убедиться, что вы правы .


Эта загадка с коленом — дьявольская. Удачи в решении.

(Сложность: 🚨HARD 🚨 )

Когда-нибудь в будущем, когда будут сняты запреты на предоставление убежища, супружеская пара Флориан и Джулия направятся в бар, чтобы отпраздновать свое новообразование. Свобода.

Там они находят еще четырех пар, которым пришла в голову такая же идея.

Стремясь к социальному контакту, каждый человек в пяти парах с энтузиазмом постукивает локтями (новое рукопожатие) каждому человеку, которого они еще не встретили .

На самом деле оказалось, что многие люди знали друг друга раньше, поэтому, когда Джулия спрашивает всех, сколько локтей они постучали, она получает девять разных ответов!

Вопрос: сколько локтей постучал Флориан?

Подсказка

Какие девять ответов услышала Джулия?

Решение

Готовы к решению? Щелкните здесь, чтобы убедиться, что вы правы .


После попытки разгадать загадку с виски вам понадобится напиток

(Сложность: легко)

Алан и Клэр живут согласно старой шотландской поговорке: «Никогда не пейте виски без вода, ни вода без виски! » Итак, однажды, когда перед Аланом стоит стакан виски, а перед Клэр стоит стакан воды такого же размера, Алан берет ложку своего виски и наливает ее в воду Клэр.

Клэр перемешивает воду с оттенком виски, а затем кладет ложку этой смеси обратно в виски Алана, чтобы убедиться, что они выпили ровно столько же.

Итак: В виски Алана больше воды, а в воде Клэр больше виски? И имеет значение, насколько хорошо Клэр перемешалась?

Подсказка

Размер ложки не имеет значения , а .

Решение

Готовы к решению? Щелкните здесь, чтобы узнать, правы ли вы.


Проблема дудла намного сложнее, чем кажется. Сможете ли вы это решить?

(Сложность: средняя)

Загадка этой недели относительно проста, но все же зловеща.

Вопрос: Можете ли вы получить 100, вставив любое количество плюсов и минусов в строку цифр 9 8 7 6 5 4 3 2 1? Вы не можете изменить порядок цифр ! Итак, какое количество плюсов и минусов должно быть у минимум , чтобы получить 100?

Эндрю Дэниэлс


Например, 98-7-6 + 54-32 показывает один способ перемежать плюсы и минусы, но, поскольку он равен 107, это не решение.

Я называю это «проблемой каракуля»: над ней лучше всего работать во время встреч, где в противном случае вы могли бы рисовать.

Подсказка

Возможно, вы захотите начать поиск решений, которые используют в общей сложности семь плюсов и минусов (хотя есть способы использовать меньше).

Решение

Готовы к решению? Щелкните здесь, чтобы узнать, правы ли вы.


Эта математическая головоломка озадачила каждого ученого, кроме одного.Думаете, вы сможете это взломать?

(Сложность: HARD)

В честь Фримена Дайсона, известного физика, который умер в прошлом месяце , вот легендарная история, демонстрирующая его сообразительность и невероятную силу ума.

Однажды на собрании ведущих ученых один из них вслух поинтересовался, существует ли целое число, которое можно было бы удвоить, переместив последнюю цифру на передний план. Например, 265 удовлетворял бы этому , если бы 526 было его точным двойником, а это не так.

После , очевидно, всего за пять секунд , Дайсон ответил: «Конечно, есть, но наименьшее такое число состоит из 18 цифр».

Это оставило некоторых самых умных ученых в мире недоумением, как он мог это выяснить так быстро.

Итак, учитывая подсказку Дайсона, какое наименьшее такое число?

Подсказка

Мой второклассник недавно научился складывать себе трехзначное число с помощью классического вертикального метода:

Эндрю Дэниэлс

18-значных чисел, конечно, можно сложить таким же образом.

Решение

Готовы к решению? Щелкните здесь, чтобы убедиться, что вы правы .


Определите, что у нее на лбу

(Сложность: средняя)

Сесилия любит проверять логику своих очень логичных друзей Джайи, Джулиана и Леви, поэтому она объявляет:

«Я напишу положительное число на каждом твоем лбу. Ни одно из чисел не совпадает, и два числа в сумме дают третье.

Она пишет цифры на их головах, затем поворачивается к Джайе и спрашивает ее, какой у нее номер. Джая видит, что у Джулиана 20 на лбу, а у Леви 30 на лбу. Она задумывается на мгновение, а затем говорит: «Я не знаю, какой у меня номер». Джулиан вставляет: «Я тоже не знаю своего номера», а затем Леви восклицает: «Я тоже!» Сесилия радостно говорит: «Я наконец-то поставила вас в тупик, ребята!»

«Не так быстро!» — говорит Джая. «Теперь я знаю свой номер!»

Какой номер у Джайи?

Подсказка

Джая может быть одним из двух чисел, но только одно из этих чисел приведет к тому, что Джулиан и Леви не будут знать своих чисел. Почему?

Решение

Готовы к решению? Щелкните здесь, чтобы убедиться, что вы правы .


Сможете ли вы добиться избрания Киану Ривза президентом?

(Сложность: средняя)

На дворе 2024 год, на демократических праймериз баллотируются пять кандидатов: Тейлор Свифт, Опра Уинфри, Марк Кьюбан, Киану Ривз и Дуэйн Джонсон. (Эй, это может случиться.) Как обычно, первые первичные выборы проходят в Айове.

Пытаясь преодолеть замешательство после фиаско фракционного собрания 2020 года, Демократическая партия Айовы только что объявила о новом, надежном способе поиска лучшего кандидата: выборы пройдут четыре раза подряд.

Сначала кандидат 1 будет баллотироваться против кандидата 2. Затем победитель этого конкурса будет баллотироваться против кандидата 3, затем этот победитель будет баллотироваться против кандидата 4, и, наконец, победитель этих выборов будет баллотироваться против последнего кандидата. Согласно переходному свойству, победитель последних выборов должен быть лучшим кандидатом… так говорит Демократическая партия Айовы.

Кандидат Киану чувствует себя довольно плохо, так как он знает, что большинство избирателей оценивают его ближе к низу, а к верхнему — нет. Фактически, он знает, что население Айовы разделено на пять равных групп и их предпочтения следующие:

.

Киану дружит с детства с Биллом С. Престоном, эсквайром, новым главой Демократической партии Айовы. Престон, уверенный, что порядок кандидатов не имеет значения для результата, говорит Киану, что он может выбирать порядок голосования кандидатов.

Итак, какой порядок выбрать Киану?

Подсказка

Как Киану будет выступать в гонках один на один против каждого кандидата?

Решение

Готовы к решению? Щелкните здесь, чтобы убедиться, что вы правы .


Кто открыл все эти проклятые шкафчики?

(Сложность: Средняя)

В главном коридоре средней школы Хелма есть 100 шкафчиков. Каждую ночь директор школы следит за тем, чтобы все шкафчики были закрыты, чтобы следующий день был организован.Однажды 100 озорных учеников решают, что разыграют.

Учащиеся собираются перед началом занятий и выстраиваются в очередь. Затем первый студент идет по коридору и открывает все шкафчики. Следующий ученик закрывает все остальные шкафчики (начиная со второго). Затем ученик 3 подходит к каждому третьему шкафчику (начиная с третьего) и открывает его, если он закрыт, и закрывает, если он открыт. Учащийся 4 открывает каждый четвертый шкафчик, если он закрыт, и закрывает его, если он открыт.Это продолжается и продолжается до тех пор, пока Студент 100 наконец не дойдет до сотого шкафчика. Когда директор приходит позже утром, какие шкафчики она находит открытыми?

Совет

Обязательно обратите внимание на все факторы.

Решение

Готовы к решению? Щелкните здесь, чтобы убедиться, что вы правы .

Этот контент создается и поддерживается третьей стороной и импортируется на эту страницу, чтобы помочь пользователям указать свои адреса электронной почты.Вы можете найти дополнительную информацию об этом и подобном контенте на сайте piano.io.

Сложных головоломок и загадок, которые могут разгадать только умные люди

12. Во время недавней переписи человек сказал переписчику, что у него три дети. Когда его спросили, их возраст, он ответил: «Произведение их возраста равно 72. Сумма их возраста совпадает с номером моего дома.«Счетчик подбежал к входной двери мужчины и посмотрел на номер дома.« Я все еще не могу сказать », — пожаловалась она. Мужчина ответил:« О, верно, я забыл сказать вам, что самый старший любит шоколадный пудинг. Затем переписчик сразу записал возраст троих детей. Сколько им лет?

Семья сидит в своей гостиной на Лонг-Айленде, Нью-Йорк, 1951 год. Эд Форд / AP

Пояснение: Как объяснили пользователи Reddit TT1103 и RedditRage, ключом к этой головоломке является то, что счетчик переписи смотрит на номер дома. Другими словами, она знает сумму возрастов детей.

Однако на этом этапе загадки она все еще не может сказать, сколько лет детям этого мужчины.Следовательно, она должна застрять между множеством возможностей. Чтобы еще больше сузить его, только два набора чисел, которые умножаются на 72, имеют одинаковую сумму: (2,6,6) и (3,3,8).

Однако после того, как мужчина сообщает, что его старшему ребенку нравится шоколадный пудинг, счетчик может различить эти два варианта. То есть только у последнего из этих двух наборов есть отдельный «старший» ребенок.

Источник: mikebrown_pelican, Reddit

Самая сложная головоломка, которую вы когда-либо видели, и секрет, который вам нужен для ее решения

Будучи докторантом Принстонского университета в 1957 году, обучаясь у основоположника теоретической информатики, Раймонд Смуллиан время от времени посещал Нью-Йорк. Во время одного из таких визитов он встретил «очень обаятельную женщину-музыканта», и на их первом свидании Смуллян, неисправимый флирт, действовал очень логично — и хитроумно.

«Не могли бы вы сделать мне одолжение?» — спросил он ее. «Я должен сделать заявление. Если утверждение верно, дадите ли вы мне автограф? »

Довольна, чтобы подыгрывать, она ответила: «Не понимаю, почему бы и нет».

«Если заявление ложное, — продолжал он, — ты не дашь мне автограф».

«Хорошо…»

Его заявление было: «Вы не дадите мне ни автографа, ни поцелуя.

Это занимает некоторое время, но со временем становится очевидным хитрость уловки Смулляна.

Правдивое заявление дает ему автограф, как они договорились. Но утверждение Смулляна, если предположить, что оно истинно, приводит к противоречию: оно исключает возможность давать автограф. Это делает заявление Смулляна ложным. А если заявление Смулляна ложное, то очаровательная женщина-музыкант подарит ему либо автограф, либо поцелуй. Теперь вы видите ловушку: она уже согласилась не награждать ложное заявление автографом.

С помощью логики Смуллян превратил ложное заявление в поцелуй. (И в прекрасный роман: они в конце концов поженятся.)

Именно такую ​​логическую игривость любит Смуллян, и все, кажется, любят его за это. Его книги по развлекательной математике и логике с названиями вроде Как называется эта книга? и «Поиздеваться над пересмешником» не только побудил людей сделать карьеру в этих темах, но и изменил способ преподавания математики и логики.За почти столетие своей жизни 96-летний Смуллян стал опытным пианистом и фокусником, внес фундаментальный вклад в современную логику и написал о даосской философии и шахматах. «Он — бесспорный мастер логических головоломок», — говорит Брюс Горовиц, один из его бывших докторов наук. студенты, сказал.

Одним из признаков наследия Смулляна является интерес философов и логиков к его самой сложной головоломке, известной как Самая сложная логическая головоломка на свете. Звание было дано философом логики из Массачусетского технологического института, коллегой Смоллиана по имени Джордж Булос, который — не сутулится — обожал логические задачи любого рода.Однажды он испытал себя, прочитав лекцию о второй теореме Геделя о неполноте, «одном из важнейших результатов современной логики», используя только односложные слова.

Самая сложная логическая головоломка в мире выглядит так:

Три бога A, B и C называются в некотором порядке: Истинный, Ложный и Случайный. Истинный всегда говорит правду, Ложь всегда говорит ложно, но говорит ли Рэндом правдиво или ложно — это совершенно случайный вопрос . Ваша задача — определить личности A, B и C, задав три вопроса «да-нет»; каждый вопрос должен быть задан ровно одному богу.Боги понимают английский, но ответят на все вопросы на своем родном языке, где слова «да» и «нет» — «да» и «я» в определенном порядке. Вы не знаете, какое слово какое означает.

Всегда готовый к трудностям, я сел на диван с ручкой и бумагой в руке, уверенный, что смогу решить головоломку за два часа. Мне казалось, что все, что мне нужно сделать, — это сразу задать три вопроса, а затем проработать их последствия. Я спросил А, например, истинно ли В; спросил B, правда ли A; и спросил C, правда ли он.Несколько часов спустя, задавая богам все вопросы «да» и «нет», которые я мог придумать, я понял, как загадка получила свое название. Ясно, что мои вопросы не заставляли богов отвечать так, как я хотел.

Разочарованный, я отправился на поиски просветления. Хозяином на вершине горы оказался Булос, решивший загадку в 1996 году. Как он это сделал, оказалось одним из лучших уроков логики и истины, которые я когда-либо получал. Если вы хотите самостоятельно решить эту головоломку, можете перестать читать здесь.Удачи! Если тебе это удастся, мои поздравления. Но если вы этого не сделаете, возвращайтесь, и вы можете обсудить решение Boolos со мной ниже.

Ye Gods: В самой сложной логической головоломке нужно определить истинные личности богов с именами Истинный, Ложный и Случайный. Де Агостини / Archivio J. Lange

Первое, что говорит нам Булос, это то, что вы не должны делать ошибка, заключающаяся в создании всех ваших вопросов сразу, как это сделал я, в надежде, что стратегия предположения и вывода окупится. Вместо этого первое, что вы должны сделать, это выяснить вопрос, который определит, кто не может быть случайным, а кто только может быть истинным или ложным.Это поможет вам раскрыть личность Рэндома путем исключения. И как только вы определили Рандом, легко разоблачить два других.

Чтобы лучше понять этот подход, говорит Булос, нам нужно увидеть, как он работает в трех более простых головоломках.

Первая простая головоломка на самом деле представляет собой повторение классических загадок Смулляна о рыцарях и мошенниках, представленных в его книге Как называется эта книга? В головоломках рыцари всегда говорят правду, мошенники всегда лгут, и ваша задача — по их словам выяснить, кто есть кто.

Итак, для нашей первой головоломки предположим, что вы не можете вспомнить, является ли Плутон карликовой планетой, и вам нужно выяснить это, спросив кого-нибудь поблизости, но вы не знаете, рыцарь ли этот человек или лжец. Какой единственный вопрос типа «да-нет» вы можете задать, чтобы выяснить, является ли Плутон карликовой планетой?

Как объясняет Булос, вы хотите заставить рыцаря или лжеца дать вам то, что вы хотите. И вы устанавливаете эту ловушку с помощью фразы тогда и только тогда, когда — логическая конструкция, называемая «двусмысленной.Итак, в этом случае ваш вопрос будет следующим: «Вы рыцарь, если и только если Плутон — карликовая планета?»

Когда вы вставляете if и only if «между двумя утверждениями, которые либо истинны, либо оба ложны, вы получаете утверждение, которое истинно; но если вы вставите его между одним истинным и одним ложным утверждением, вы получите ложное утверждение », — пишет Булос. Это похоже на знак умножения: так же, как вставка знака умножения между двумя положительными или двумя отрицательными числами дает вам число, которое всегда положительно, вставка тогда и только тогда, когда между двумя истинными или двумя ложными утверждениями дает вам утверждение, которое всегда правда.

Учитывая, что вы могли бы адресовать свой вопрос рыцарю или лжецу, есть четыре возможных ответа (при условии, что мы не знаем, что Плутон на самом деле карликовая планета):

1. Если человек рыцарь а Плутон — карликовая планета, тогда вы получите ответ «да», поскольку оба утверждения по обе стороны от тогда и только тогда, когда верны, а рыцари всегда говорят правду.
2. Если человек рыцарь, а Плутон не карликовая планета, вы получите «нет», поскольку вопрос содержит ложное утверждение.
3. Если человек лжец, а Плутон — карликовая планета, вы получите «да», поскольку лжецы всегда говорят ложно, и правильный ответ — «нет».
4. Если человек лжец, а Плутон не карликовая планета, вы получите «нет», поскольку правильный ответ — «да».

Посмотрите, что только что произошло: формулируя вопрос с двояким условием, вы получаете желаемую информацию — если Плутон — карликовая планета, вы получите ответ «да», а если нет, вы получите «нет» — будь вы говорить с кем-то честным или нет. Однако помните, что в Самой сложной логической головоломке рыцарь и лжец — в отличие от этого примера — не говорят по-английски.

Чтобы заманить их в ловушку, чтобы они рассказали, означают ли «да» и «джа» «да» и «нет» или наоборот, рассмотрим вторую простую загадку Булоса.

В этой загадке вы знаете, что спрашиваете рыцаря, который всегда говорит правду, но он отвечает только «да» и «джа». Какой один вопрос типа да-нет вы можете задать, чтобы выяснить, является ли Плутон карликовой планетой? Исходя из последней загадки, вы получили следующее: спросите рыцаря: «Значит ли« да »« да »тогда и только тогда, когда Плутон — карликовая планета?» Бинго: вы получите ответ «да», если Плутон — карликовая планета, и «джа», если нет, даже если мы не знаем, что означают эти слова.Результат такой же прибыльный, как и приведенный выше: незнание того, был ли человек рыцарем или лжецом, не было препятствием, так же как и незнание значений «да» и «джа».

Boolos сообщает нам, что самая сложная логическая головоломка — это, по сути, две первые простые головоломки плюс третья. Однако, прежде чем мы это решим, посмотрим, как сочетаются первые две головоломки: вам нужно выяснить, является ли Плутон карликовой планетой, и вы должны спросить кого-нибудь, кто может быть рыцарем или лжецом и , он ответит отвечайте только «да» или «я».«Какой вопрос вы бы задали? Если вы думаете, что раз уж это сложная головоломка, то правильно задать сложный вопрос, то вы правы! Спросите: «Означает ли« да »« да »тогда и только тогда, когда вы рыцарь, и тогда и только тогда, когда Плутон — карликовая планета?» Задав этот вопрос, вы получите ответ «да», если Плутон — карликовая планета, и «джа», если нет, независимо от того, обращаетесь ли вы к рыцарю или лжецу. Этот хорошо сформулированный вопрос подобен ключу от замка.

Каждое утверждение либо истинно, либо ложно — золотой середины нет.

А вот и третья простая головоломка. Его правила таковы. Предположим, я кладу перед вами три карты в ряд — два туза и валет — лицом вниз. Вы не знаете, как они заказаны, но я знаю. Задав мне один вопрос типа «да-нет», указав на одну из карт, вы сможете с уверенностью идентифицировать одну из карт как туза. Если вам случится указать на одного из двух тузов, я отвечу на вопрос честно, как рыцарь; если вместо этого вы укажете на валета, я отвечу «да» или «нет» наугад, как случайный бог.Куда вы укажете и что спросите?

Это может показаться немного сложнее, но на самом деле это не так. Укажите на любую карту и спросите, является ли одна из других карт тузом. Допустим, вы указываете на среднюю карту и спрашиваете, является ли левая карта тузом. « Независимо от того, является ли средняя карта тузом или нет , — говорит Булос, — вы обязательно найдете туз, выбрав левую карту, если вы слышите, как я говорю« да », и выбрав правильную карту, если вы услышите« нет ». » Почему? Ну, если средняя карта — туз, тогда, когда я говорю «да», левая карта тоже туз; если я говорю «нет», то правильная карта.Если средняя карта — валет, не имеет значения, скажу я случайным образом «да» или «нет»: левая и правая карты должны быть тузами, поскольку средняя — валет. Итак, указали вы на туза или нет, мой ответ на ваш вопрос, «да» или «нет», всегда будет определять местонахождение другого туза, если карта, о которой вам интересно, не та. вы указываете на.

Указывать на любую карту и спрашивать о личности другой — это стратегия отлова, которую вам нужно адаптировать, чтобы выяснить, кто из должен быть истинным или ложным в самой сложной логической головоломке.В рамках вашего вопроса вы переводите указание на любую карту в слова, чтобы заменить фактическое утверждение «Плутон — карликовая планета» в составной головоломке выше утверждением , кто такое Random — что вы Заметьте, решение является столь же произвольным, как и на какую карту указать. Кому мы говорим, что случайность находится в вопросе, будет зависеть от того, кому мы решим задать вопрос. Это не имеет значения; это мог быть любой из трех богов.

Давайте зададим вопрос богу A и утвердим, что B является случайным: «Означает ли« da »« да »тогда и только тогда, когда вы истинны, тогда и только тогда, когда B является случайным?» Это равносильно тому, чтобы указать на B, задавая вопрос о личности A. В карточной головоломке независимо от того, ответил ли я правдиво или случайно на , вы можете положиться на мой ответ «да» или «нет», чтобы с уверенностью найти туз. То же самое и здесь. « Независимо от того, истинно ли A, ложно или случайно, , — говорит Булос, — если вы получите ответ da, C будет либо True, либо False, а если вы получите ответ ja, B будет либо True. или Ложь! »

Предположим, у нас есть «джа» (мы должны предположить одно или другое). Это делает B либо истинным, либо ложным, что именно то, что мы хотели — мы уже знаем, как разоблачить кого-то вроде этого: спросите B: «Означает ли« да »« да », если и только если Плутон — карликовая планета?» Поскольку мы знаем, что Плутон на самом деле является карликовой планетой, есть два возможных ответа:

1.Если B верно, то вы получите ответ «да».
2. Если B — False, то вы получите «ja», поскольку правильный ответ — «da», а False всегда говорит ложно.

Предположим, у нас есть «da», что делает B истинным. Теперь задайте True ваш третий и последний вопрос: «Означает ли« да »« да »тогда и только тогда, когда А является случайным?» Учитывая, что Random должно быть A или C, возможен только один ответ:

1. Поскольку B имеет значение True, вы получаете «da», что означает, что A является случайным, и, следовательно, C является ложным.

Подведем итоги: используя всю логику Булоса, наши три вопроса, чтобы определить, какой бог является Истинным, Ложным и Случайным, будут выглядеть следующим образом:

1.Богу А: «Означает ли« да »« да »тогда и только тогда, когда ты Истинен, и тогда и только тогда, когда В — Случайно?» (Мы предположили, что А сказал «джа», что сделало Б истинным или ложным).
2. Богу B: «Означает ли« да »« да »тогда и только тогда, когда Плутон — карликовая планета?» (Мы предположили, что B сказал «да», что сделало B истинным.)
3. И снова для бога B (истинно): «Означает ли« da »« да »тогда и только тогда, когда A является случайным?» Поскольку B истинно, он должен сказать «да», что означает, что A является случайным, а C остается ложным.

Решено!

Так чему же нас учит самая сложная логическая головоломка? Согласно Булосу, это показывает нам, насколько важным кажется один из предполагаемых фундаментальных законов логики — закон исключенного третьего.«Наша способность рассуждать об альтернативных возможностях, — говорит Булос, — даже в повседневной жизни, была бы почти полностью парализована, если бы нам было отказано в применении закона исключенного третьего». Закон исключенного третьего просто таков: каждое утверждение либо истинно, либо ложно — золотой середины нет. Это отрезвляющая мысль. Но мы должны только поблагодарить Смулляна, эту самую дьявольскую загадку, за то, что он заставил нас задуматься над ней с таким назидательным восторгом.

Брайан Галлахер — помощник редактора в Nautilus.

сложных логических головоломок

1. Самый умный принц

Король хочет, чтобы его дочь вышла замуж за самого умного из трех очень умных молодых принцев, и поэтому мудрецы короля разработали тест на интеллект.

Принцы собираются в комнате и сидят лицом друг к другу; им показаны 2 черные шляпы и 3 белые шляпы. Им завязывают глаза, и на каждую из них надевают по одной шляпе, а остальные шляпы спрятаны в другой комнате.

Король говорит им, что первый принц, который определит цвет его шляпы, не снимая ее и не глядя на нее, женится на его дочери. Неправильная догадка будет означать смерть. Затем повязки снимаются.

Вы один из принцев. Вы видите 2 белые шляпы на головах другого принца. Через некоторое время вы понимаете, что другие принцы не могут определить цвет своей шляпы или не хотят угадывать. Какого цвета твоя шляпа?

Примечание: вы знаете, что ваши конкуренты очень умны и не хотят ничего, кроме как жениться на принцессе.Вы также знаете, что король человек слова, и он сказал, что это испытание — это честное испытание ума и храбрости.

Подсказка: Исходя из того, что вы знаете, почему другие принцы не могут решить эту головоломку?

Раствор: белый.

Король не выбрал две белые шляпы и одну черную шляпу. Это означало бы, что два принца увидят одну черную шляпу и одну белую шляпу. Если бы вы были единственным принцем в черной шляпе, вы оказались бы в невыгодном положении.

Если бы вы носили черную шляпу, одному из других принцев не потребовалось бы много времени, чтобы сделать вывод, что он носит белую шляпу.

Если бы разумный принц увидел белую шляпу и черную шляпу, он в конце концов понял бы, что король никогда не выберет две черные шляпы и одну белую шляпу. Любой принц, увидевший две черные шляпы, сразу узнал бы, что на нем белая шляпа. Следовательно, если принц видит одну черную шляпу, он может решить, что он одет в белую.

Следовательно, единственное честное испытание для всех трех князей — быть в белых шляпах.Подождав некоторое время на всякий случай, вы можете смело утверждать, что на вас белая шляпа.


2. 100 золотых монет

Пятеро пиратов получили 100 золотых монет и должны поделить добычу. Все пираты чрезвычайно умны, коварны и эгоистичны (особенно капитан).

Капитан всегда предлагает распределение добычи. Все пираты голосуют за предложение, и если половина экипажа или более ответят «Да», добыча будет разделена, как предлагается, поскольку ни один пират не захочет сразиться с капитаном без превосходящей силы на их стороне.

Если капитану не удается заручиться поддержкой хотя бы половины своей команды (включая его самого), ему грозит мятеж, и все пираты восстанут против него и заставят его идти по доске. Пираты начинают сначала со следующим старшим пиратом в качестве капитана.

Какое максимальное количество монет капитан может оставить, не рискуя своей жизнью?

Подсказка: что будет, если пиратов два? Кто полностью проигрывает? Что будет, если пиратов будет три? Кто полностью проигрывает? Что будет, если пиратов будет четыре? Какие два пирата полностью проигрывают?

Решение: 98

Капитан говорит, что возьмет 98 монет и отдаст одну монету третьему по старшинству пирату, а вторую — самому младшему пирату. Затем он объясняет свое решение следующим образом …

Если бы было 2 пирата, из которых пират 2 был бы самым старшим, он бы просто проголосовал за себя, и это было бы 50% голосов, так что он, очевидно, оставит все деньги себе.

Если пиратов было 3, пират 3 должен убедить хотя бы еще одного человека присоединиться к его плану. Пират 3 возьмет 99 золотых монет и отдаст 1 монету пирату 1. Пират 1 знает, что если он не проголосует за пирата 3, то он ничего не получит, поэтому, очевидно, он будет голосовать за этот план.

Если бы пиратов было 4, пират 4 дал бы 1 монету пирату 2, а пират 2 знает, что если он не проголосует за пирата 4, то он ничего не получит, поэтому, очевидно, он будет голосовать за этот план.

Так как пиратов 5, пиратам 1 и 3 явно лучше голосовать за капитана, иначе им придется ничего не выбирать или рисковать смертью.


3. 1 золотая монета

К пяти упомянутым ранее пиратам присоединяется шестой, а затем он грабит корабль, имея только одну золотую монету.

После того, как они выразили свое разочарование, убив всех на борту корабля, им теперь нужно разделить одну монету. Они настолько разгневаны, что теперь ценят их в порядке приоритета:
1. Свои жизни
2. Получение денег
3. Видеть, как умирают другие пираты.

Итак, если бы им был дан выбор между двумя исходами, в которых они получали одинаковую сумму денег, они бы выбрали исход, при котором они увидят, как погибнет больше других пиратов.

Как капитану спасти свою шкуру?

Подсказка: используйте тот же подход.

Решение: Самый старший пират может отдать монету младшему пирату. Он может использовать ту же логику в предыдущей головоломке, чтобы объяснить тщетность попытки оставить монету себе.


4. Греческие философы

Однажды три греческих философа поселились в тени оливкового дерева, открыли бутылку Рецины и начали длительное обсуждение фундаментального онтологического вопроса: почему что-то существует?

Через некоторое время они начали бродить. Затем один за другим они заснули.

Пока мужчины спали, три совы, по одной над каждым философом, завершили свой пищеварительный процесс, уронили каждому философу подарок на лоб, и тот улетел с шумным «улюлюканьем».

Возможно, крик разбудил философов. Как только они посмотрели друг на друга, все трое одновременно рассмеялись. Затем один из них внезапно перестал смеяться. Почему?

Подсказка: Тот, кто перестал смеяться, спросил себя, что видят другие философы, что заставляет их смеяться.

Решение: Если бы у него (самого умного философа) ничего не было на голове, тогда он понял бы, что второй умнейший философ быстро понял бы, что третий умнейший смеется только над вторым умнейшим философом, и, следовательно, второй умнейший философ будет перестал смеяться.


5. 100 монет

Есть 10 наборов по 10 монет. Вы знаете, сколько должны весить монеты. Вы знаете, что все монеты в одном наборе из десяти имеют отклонение ровно на одну сотую унции, что делает весь набор из десяти монет на одну десятую унции.Вы также знаете, что все остальные монеты имеют правильный вес. Вы можете использовать чрезвычайно точные цифровые весы только один раз.

Как определить, какой набор из 10 монет неисправен?

Подсказка: вы можете взвесить столько или меньше из десяти монет из каждого набора, сколько захотите.

Решение: Одна монета из первого набора кладется на весы вместе с двумя монетами из второго набора и т. Д… Если вес отклонен на одну сотую унции, то это первый набор неисправен, если вес отклонен на две сотни унций, то это второй набор, который неисправен, и т. Д …


6. Обезьяна и кокос

Десять человек высаживаются на необитаемый остров. Там они находят много кокосов и обезьяну. В первый день они собирают кокосы и складывают их все в общую кучу. После целого дня работы они решают поспать и на следующее утро разделить их на десять равных куч.

В ту ночь один из потерпевших просыпается голодным и решает пораньше принять свою долю. Разделив кокосы, он обнаруживает, что на один кокос не хватает десяти равных куч. Он также замечает обезьяну, держащую еще один кокосовый орех. Поэтому он пытается взять кокос обезьяны, чтобы общая сумма делилась на 10. Однако, когда он пытается взять его, обезьяна бьет его им по голове и убивает.

Позже другой потерпевший просыпается голодным и решает рано забрать свою долю.По пути к кокосам он находит тело первого потерпевшего крушение, что ему нравится, потому что теперь он получит право на 1/9 всей стопки. Разделив их на девять стопок, он снова оказался на один кокос меньше и пытается взять слегка окровавленный кокос обезьяны. Обезьяна бьет второго человека по голове и убивает его.

Один за другим каждый из оставшихся потерпевших крушение проходит один и тот же процесс, пока 10-й проснувшийся человек не получит всю стопку для себя. Какое наименьшее количество кокосовых орехов в куче, не считая обезьян?

Подсказка: найдите формулу НОК.

Решение: 2519

Ответом является НОК (наименьшее общее кратное) 10,9,8,7,6,5,4,3,2,1 -1. НОК даст наименьшее число, которое делится на все эти числа, а вычитание единицы даст нам количество кокосов, которые изначально были там.


7. Подбрасывание монет

На столе лежит двадцать монет, десять — решка, а десятки — решка.Вы сидите за столом с завязанными глазами и в перчатках. Вы можете чувствовать, где находятся монеты, но не можете увидеть или почувствовать, орел они или решка. Вы должны создать два набора монет. В каждом наборе должно быть такое же количество орлов и решек, что и в другой группе. Вы можете только перемещать или подбрасывать монеты, вы не можете определить их текущее состояние. Как создать две четные группы монет с одинаковым количеством орлов и решек в каждой группе?

Решение: Создайте два набора по десять монет. Переверните монеты в одном из наборов и оставьте монеты в другом наборе. Первый набор из десяти монет будет иметь такое же количество орлов и решек, что и другой набор из десяти монет.


8. Двое детей

Я наугад спрашиваю людей, есть ли у них двое детей, а также мальчик, родившийся во вторник. После долгих поисков я наконец нашел того, кто ответил утвердительно. Какова вероятность того, что у этого человека будет два мальчика? Предположите равные шансы родить обоих полов и равные шансы родить в любой день.

Решение: 13/27. Если вы думаете, что ответ должен быть 1/2, вы ошибаетесь. Если бы вы знали, какой ребенок был мальчиком (скажем, младший), вы были бы ближе к истине. Но поскольку мальчик может быть либо младшим, либо старшим ребенком, анализ более тонкий. Но при чем тут вторник?


.

Добавить комментарий

Ваш адрес email не будет опубликован. Обязательные поля помечены *